NNP Boards Random Factos to Remember

Pataasin ang iyong marka sa homework at exams ngayon gamit ang Quizwiz!

Cannalicular stage 16-26wks

. Formation of gas-exchanging units and respiratory tree that supplies air to the lungs b. Capillaries invade the airway walls and form the first air-blood barrier c. Epithelial cells appear d. Airway changes from glandular to tubular e. Respiratory bronchioles can be differentiated f. Have some gas exchange capacity g. Beginning of alveolar Type II epithelial cell differentiation to produce surfactant proteins

Embryonic stage wks 1-5

. The embryonic foregut provides a single lung bud that begins to divide b. Pulmonary vein develops and joins the lung bud c. The trachea develops at the end of this stage d. Lobes of the lung begin to develop as well

Prostaglandins

0.05-0.4mcg/kg/min relaxes smooth muscle SE: apnea, bradycardia, hypotension, fever, thrombocytopenia see results in 30min

TPN mag

0.3-0.6mEq/kg

TPN phos

1-1.15mM/kg (Ca to phos ration is 2:1)

Acetate

1-2 mEq/kg/day

Transepidermal water loss in the extremely low birthweight preterm infant can be how much higher than in the term infant? 10-15 fold higher 25-30 fold higher 45-50 fold higher

10-15

Normal saline is how many mEq/L

154mEq/L = 0.154mEq/ml

18q21.2

18q21.2 Chromosome 18, long arm, region 21, band 2

TPN K

2-4 mEq/kg

TPN calcium

2-4 mEq/kg

TPN sodium

2-4 mEq/kg

In necrotizing enterocolitis what percentage of blood cultures may be positive? 60-70% 40-50% 20-30%

20-30%

Heart starts to beat on day

22

how much fluid is excreted from the fetal pulmonary system?

300-400ml

GFR is delayed until

34 weeks mature at 1-2 years

Newborn sweat glands function poorly before which gestational age? 28 weeks 32 weeks 36 weeks

36

1/4 NS is how many mEq/L

37mEq/L = 0.037mEq/mL

The stratum corneum develops during which trimester? First Second Third

3rd

primordial gut forms during the

4th week

For a single gene autosomal dominant disorder, the risk of inheriting the mutated allele from the affected parent is: 25% 50% 100%

50%

Recommended duration of treatment with Zidovudine (AZT, ZDV) is: 1 week 6 weeks 3 months

6 weeks

For best results, Zidovudine (AZT, ZDV) should be started in the newborn born to an HIV infected mother by: 1 hour of life 6 to 12 hours of birth 24-48 hours of life

6-12hrs

Nephrogenesis begins at

7-8wks and is completed at 34wks

One gram of albumin can bind with approximately how much bilirubin? 1 mg 5 mg 8 mg

8

what gestational age can you see swallowing?

8-11 weeks

when are renal nephrons formed?

9-11 weeks

Average weight gain

<1 kg 10-15 gm/kg/day ii. 1-2 kg 15-20 gm/kg/day iii. >2 kg 25-35 gm/day

Peripheral mOsm

<900

The perception or attitude about cause and effect that may skew a study's design or outcome is known as: Bias An attribute variable The Hawthorne effect

A bias is an attitude, opinion, or personal perception concerning a cause-and-effect relationship that may skew study results.

The classification of hydrocephalus as communicating or non-communicating is based on the: Anatomical location of the obstruction Associated anomalous structures Rapidity and volume of ventricular enlargement

A blockage in the ventricular system that prevents CSF from entering the subarachnoid space causes non-communicating hydrocephalus (e.g., aqueductal stenosis, atresia of the foramen of Monro, intraventricular neoplasm). A blockage outside the ventricular system that prevents the absorption into the venous system (such as meningitis) causes communicating hydrocephalus

The researcher tests the stated relationship between the identified variables based on the: Hypothesis Null hypothesis Population sample

A hypothesis is a formal statement of expected or predicted relationships between two or more variables in a specified population or group. On the basis of the hypothesis, the researcher tests the stated relationship between the identified variables

In comparison to primary maternal CMV infection, non-primary maternal CMV infection (re-infection): Has a higher likelihood of causing congenital disease in the neonate Has a higher CMV transmission rate Has an increased risk of teratogenicity in late pregnancy

A non-primary maternal CMV infection (reinfection) during pregnancy has a lower transmission rate than a primary infection, but it is responsible for over 85% of the burden of congenital CMV infection. Infection or reinfection during early pregnancy has teratogenic potential.

An example of a nonrandom occurrence of multiple malformations for which no common etiology has been identified is: VACTERL Von Willebrand disease Pierre Robin

A nonrandom occurrence of multiple malformations for which no common etiology has been identified is an association. An example is VACTERL association. Pierre Robin is a sequence and von Willebrand disease is a coagulopathy transmitted most often in an autosomal dominant manner.

The following compete blood count (CBC) was obtained from a 12-hour old term neonate. WBC 20,500 / mm3, Neutrophils 36%, Bands 4%, Lymphocytes 35%, Monocytes 18%, Eosinophils 5%, Basophils 2%. These results are: Normal for age Shifted to the left Shifted to the right

A normal WBC count in a neonate is 5000- 30,000 cells/mm3, I/T ratio for this infant is 0.1 (I/T ratio of greater than 0.20 is suggestive of infection)

A 1 hour old neonate is found to be pale, tachycardic and with a very boggy, edematous scalp and neck. These signs are consistent with a: Subependymal hemorrhage Subarachnoid hemorrhage Subgaleal hemorrhage

A subgaleal hemorrhage is located between the scalp (galea) and the cranial periosteum. The potential space for blood to accumulate extends from the orbital ridges to the nape of the neck. A major portion of the neonate's blood volume can potentially be contained in this space, leading to severe hypovolemic shock and anemia. Clinical signs for early identification are an expanding boggy scalp, pallor, prolonged capillary refill, tachycardia, decreased responsiveness and spontaneous activity, a falling hematocrit, and signs of frank or impending shock.

Neonates receiving Acyclovir need laboratory monitoring for: Sodium Unconjugated hyperbilirubinemia Neutropenia

Acyclovir can cause acute renal injury liver changes, neutropenia, thrombocytopenia and other hematologic abnormalities. Monitoring should include CBC, BUN, creatinine, liver transaminases and a complete blood count to monitor neutrophil and platelet levels.

Alagille syndrome as a cause of conjugated hyperbilirubinemia is caused by a paucity of: Intrahepatic bile ducts Extrahepatic bile ducts Bile producing hepatocytes

Alagille syndrome is an autosomal dominant genetic disorder that has a variable expression, but has as its most consistent feature a paucity of intrahepatic bile ducts which causes cholestatic jaundice.

In an infant with necrotizing enterocolitis with perforation, which antibiotic would offer anaerobic coverage? Linezolid Metronidazole Cefotaxime

Anaerobic coverage should be added if bowel necrosis or perforation is suspected. Metronidazole is effective against anaerobic organisms.

The risk of seizure activity in an anencephalic infant is: High Low None

Anencephaly is caused by failure of the anterior neural tube to fuse in the cranial area. Because fusion of the neural tube in this area forms the forebrain, anencephalic infants have minimal development of brain tissue. The brain tissue that does develop is poorly differentiated and becomes necrotic with exposure to amniotic fluid. Thus, without effective brain tissue the risk of seizure activity is none

What percentage of infants born to mothers with active, untreated syphilis are asymptomatic at birth? 66% 40% 25%

Approximately 2/3 (66%) of liveborn infants with congenital syphilis are asymptomatic at birth.

The malformation associated with myelomeningocele which can lead to obstruction of the foramen magnum is: Absence of corpus callosum Arnold-Chiari Encephalocele

Arnold-Chiari malformation, also known as Chiari II (inferior displacement of the medulla and fourth ventricle into the upper cervical canal and elongation and thinning of the upper medulla and lower pons) is thought to result in hydrocephalus from aqueductal stenosis and blockage of the cerebrospinal fluid outflow from the fourth ventricle.

An exchange transfusion may cause: Decreased platelet count Increased platelet count No change in platelet in platelet count

Because the blood used for exchange transfusions is generally deficient in platelets it may cause a low platelet count. Platelets may decrease >50% following a transfusion. A platelet count should be done following an exchange transfusion.

The best results to Varicella Zoster Immune Globulin (VZIG) given to a neonate exposed to Varicella Zoster infection occur when it is given within: 96 hours after exposure 120 hours after exposure 168 hours following exposure

Best results are obtained when VZIG (or VariZig) is given within 96 hours of exposure.

One gram of heme produces how much bilirubin? 18 mg 26 mg 34 mg

Bilirubin is a metabolic end-product of erythrocyte breakdown. One gram of heme produces 34 mg of bilirubin, with normal newborns producing 8-10 mg/kg/day.

In comparison to adult bilirubin production, the production of bilirubin in a the first day of life in a newborn is: About the same Five times as high Twice as high

Bilirubin production is two to three times as high in the first day of life than in an adult.

Staining and necrosis of the neurons by bilirubin generally spares the: Cerebral cortex Cerebellum Basal ganglia

Bilirubin staining and necrosis of neural structures includes the basal ganglia, hippocampal cortex, subthalamic nuclei and cerebellum, generally sparing the cerebral cortex.

Bilirubin absorbs light maximally in the: Orange range (420 to 600 nm) Blue range (340 to 540 nm) White range (380 to 720 nm)

Blue lights labeled F20 T12/B or BB, or TL52 is most effective.

A potential problem related to rapid administration of phenytoin (Dilantin) is: Bleeding Cardiovascular collapse Hypertension

Cardiovascular affects can be arrhythmias, hypotension, or cardiovascular collapse with too rapid an infusion

The changes in brainstem auditory evoked response (BAER) due to hyperbilirubinemia can occur with: Mild unconjugated hyperbilirubinemia (5-9 mg/dL) Moderate unconjugated hyperbilirubinemia (10-20 mg/dL) Severe unconjugated hyperbilirubinemia (>20 mg/dL)

Changes in BAER can occur with moderate hyperbilirubinemia (10-20 md/dL), but these changes reverse with decline in the total bilirubin concentration.

Chlamydia trachomatis conjunctivitis treatment consists of: Ophthalmic erythromycin Ophthalmic + oral erythromycin Oral erythromycin

Chlamydia trachomatis treatment is erythromycin given orally 50 mg/kg/day divided into 4 doses. Topical (ophthalmic) treatment is ineffective and unnecessary.

Kernicterus can result in cerebral palsy that is: Chorioathetoid Dyskinetic Spastic diplegic

Chorioathetoid cerebral palsy is part of the classic triad of kernicterus sequelae, which also includes upward gaze and sensorineural hearing loss

Gram positive

Coagulase-negative staphylococcus ii. Staphylococcus aureus iii. Listeria monocytogenes iv. Streptococcus pneumoniae

Talipes Calcaneovalgus

Common in females May be associated with breech deliveries, hip dysplasias, and in utero positioning Treatment my include exercises and casting

Pseudogladular stage weeks 6-16

Conducting airways formed b. Main bronchi are formed c. Capillary beds are formed with blood supply d. Lung will undergo 14 more generations of branching and form the terminal bronchioles e. Form pattern corresponding to adult lung

Conjugated hyperbilirubinemia is secondary to abnormal: Bile secretion or biliary flow Bilirubin binding Conjugation of the bilirubin molecule

Conjugated hyperbilirubinemia is secondary to insufficiency of biliary secretion or bile flow or both. It is always pathologic.

What is the most commonly occurring type of Tracheoesophageal fistula (TEF)? Esophageal atresia with proximal TEF Esophageal atresia with distal TEF TEF without atresia

Correct answer is "Esophageal atresia with distal TEF". This type occurs about 85%. It also has a blind pouch for the upper esophagus.

Which assessment finding is consistent with Coarctation of the Aorta? Blood pressure gradient of greater than 15 mmHg between upper and lower extremities Loud systolic ejection click heard at apex of heart General cyanosis

Correct answer is: "blood pressure gradient of > 15 mmHg between upper and lower extremities". The narrowing of the aorta at the transverse arch causes blood pressure and perfusion to be lower in the lower extremities. The BP is the most consistent factor in critical coarctation of the aorta and is present in 97% of all cases.

What is an acid in TPN for premature infants

Cysteine

A neonate with Dandy-Walker malformation may present with which physical finding: A cloverleaf shaped skull Frontal bossing Occipital prominence

Dandy-Walker malformation consists of dilation of the fourth ventricle, agenesis of the corpus callosum and enlargement of the posterior fossa. Neonates with Dandy-Walker often have occipital enlargement or prominence.

Skin Development

Differentiation occurs from 5-26 weeks b. The skin functions include: i. Continually replenishing itself ii. Provide protective barrier iii. Repair tissue injuries c. 18-19 wks- SC is just beginning to form around hair follicles d. 16-23 wks- 4-5 epidermal layers are present e. 23-25 wks- head to toe barrier formation occurs in conjunction with hair follicle cornification f. 23 wks- the SC has terminally differentiated, but only few layers thick g. 26 wks- epidermis full keratinized h. 34-35 wks- barrier well formed

In performing the doll's eye maneuver with an infant, a normal response to head rotation would for the eyes to: Deviate away from the direction of rotation Look straight ahead Move toward the direction of the rotation

Doll's eye maneuver tests the oculomotor, trochlear and abducens nerves (Cranial nerves III, IV and VI). When the head is gently rotated, the eyes should deviate away from the direction of the rotation. If the head is rotated to the right, the eyes should turn to the left.

Gram negative

E. coli, klebsiella, serratia, enterobacter, proteus, citrobacter, pseudomonas, h flu

Acute bilirubin encephalopathy may be: Both permanent or transient Permanent Transient

Early bilirubin encephalopathy may be transient and reversible.

Early onset sepsis in the neonate is best described as: Occurs in the first 24 hours of life Occurs in the first 3 days of life Occurs in the first month of life

Early onset sepsis is occurs at or before 72 hours of age. Late onset sepsis occurs past 72 hours of age

On chest x-ray, massive cardiomegaly may suggest: Tetralogy of Fallot Ebstein's anomaly Truncus arteriosus

Ebstein's

What is the most important property of a drug?

Effectiveness

Informed consent for participation in a study should include information about all but: Consequences of refusal to participate Deviation from standard of care Confidentiality protections

Elements of informed consent are very detailed in terms of study purpose and procedure, deviation from standard of care, protection of confidentiality, costs, etc. However, no coercion for participation may be used, therefore there are no consequences for refusal to participate

A term newborn born in July is transferred to the NICU for development of a fever and poor feeding right before being discharged at 2 days of age. In addition the admission exam reveals that the infant is lethargic, appears severely ill, has a macular rash with scattered petechiae and hepatomegaly. You suspect: Enterovirus Rotavirus Herpes simplex virus

Enterovirus, more common in the summer and fall, presents with fever, poor feeding, lethargy, and the infant may have a macular or maculopapular rash with petechiae and hepatomegaly. An infant with fulminant enterovirus presents at 2-5 days of age. Mortality from hepatitis or coagulopathy can be as high as 80%.

You are called to assess a post-date, 4175 gram infant who was admitted for observation following a difficult delivery. The infant is noted to hold his left arm adducted and internally rotated, with extension at the elbow, pronation of the forearm and flexion of the wrist. The most likely cause for this is: Klumpke's palsy Erb's palsy Complete brachial paralysis

Erb's palsy is a form of brachial plexus injury. The major types of brachial plexus injuries are Erb's palsy, or upper plexus injury involving C5 to C7, and Klumpke's palsy, or lower plexus injury at C5 to T1. With Erb's palsy, the shoulder and upper arm are involved, and denervation of the deltoid, supraspinous, biceps, and brachioradialis muscles occurs. The arm lies passively at the infant's side, adducted and internally rotated, and the forearm is pronated. The grasp remains intact, but the Moro reflex is absent on that side.

A Wright stain demonstrating numerous eosinophils is consistent with which of the following: Erythema toxicum Salmon patches Infantile acne

Erythema toxicum

Polydactyly

Extra digits Often bilateral May be associated with other syndromes Treatment dependent on defect

A false positive reaction to a maternal nontreponemal test can be caused by: Neural tube defect in the fetus Maternal SSRI use Autoimmune disease

False-positive reaction to a nontreponemal antibody tests such as a VDRL or RPR can be caused by maternal autoimmune disease, intravenous drug addiction, aging, pregnancy, and many infections.

Fluconazole interferes with the metabolism of: Furosemide Digoxin Phenobarbital

Fluconazole interferes with the metabolism of barbiturates, theophylline, midazolam, phenytoin and zidovudine

Which drug can be used in combination with Amphotericin B to increase efficacy of eradicating fungal meningitis?

Flucytosine

The risk for aminoglycoside-related ototoxicity is increased with concurrent use of: Phenobarbital Furosemide Ampicillin

Furosemide and aminoglycoside antibiotics are both ototoxic, so caution should be used in using both medications in a patient

syndactyly

Fusion of 2 or more digits May be associated with other syndromes or genetic Treatment dependent on defect

Intravenous (IV) methylene blue is the first-line treatment agent for methemoglobinemia, but it is contraindicated in patients with

G6PD deficiency

common causes of neonatal infection

GBS, E. coli, listeria

Early onset bugs

GBS, E. coli, listeria, hflu

The minimum length of therapy with Ganciclovir or Valganciclovir for congenital CMV is: 2 weeks 4 weeks 6 weeks

Ganciclovir (IV) or Valganciclovir (PO) treatment of CMV must be given for a minimum of 6 weeks

Osteogenesis Imperfecta Type 1

Genetic gene mutation resulting in abnormal collagen type 1 Involves skeletal dysplasias and connective tissue disorders Six main types with varying clinical presentations Type 1 i. Most common, Growth near normal, Macrocephaly, Fractures after birth, Bowing of femur and tibia, Scoliosis, Osteopenia, Blue sclera, Hearing impairment, Triangular facies

With maternal Hepatitis C virus infection: Girls are more likely to be infected than boys There is a safe viremic level below which transmission to the fetus will not occur Any maternal rupture of membranes increases risk of neonatal infection

Girls are twice as likely to be infected as boys. Rupture of membranes greater than 6 hours increases risk of infection. There is no safe viremic level at which risk of mother to fetus transmission does not occur

Designing the research sample to reflect those who will ultimately benefit from the research results is an example of: Justice Beneficence Non-maleficence

Groups that bear the burden for research participation should be groups that will ultimately benefit from the research results.

Hyperalertness is consistent with which Sarnat Stage in a neonate with Hypoxic-Ischemic Encephalopathy? Stage 1 Stage 2 Stage 3

HIE severity is staged on the Sarnat Scale. Stage 1 is reflective of mild encephalopathy and is characterized by hyperalertness, normal muscle tone, weak suck, low threshold Moro, pupillary dilation and absence of seizures.

Which viral infectious organism should be considered in a term 10 day old term neonate presenting to the emergency department with a fever and irritability? Adenovirus Rotavirus Herpes simplex virus

HSV

A 3.6 kg term newborn with ABO incompatibility is placed under high intensity phototherapy. His bilirubin continues to rise. You order IVIG. The appropriate total dose is: 0.75 grams 1.2 grams 3.6 grams

High dose IVIG 500 mg-1000 mg/kg/dose (which equals 0.5mg-1gm/kg/dose) is given over 2-4 hours. For a 3.6 kg infant the dose would be 1.8-3.6 grams.

talipes equinovarus (clubfoot)

Hindfoot deformity More common in males Can be associated with syndromes; neural tube defects, myelomeningocele, congenital hip dysplasia May be congenital, positional, or teratogenic Treatment may include casting, surgery, or bracing

Subchorionic hemorrhage and smoker?

IUGR

In an autosomal recessive disorder, if both parents are carriers of the mutated allele, the risk of offspring having the disorder is: 25% 50% 100%

If both parents are a carrier of a mutated allele for the same autosomal recessive disorder, the risk of the offspring having the disorder is 50%

A neonate with Congenital Myotonic Dystrophy: Has a uniformly fatal outcome May improve in childhood Rarely has associated cognitive delay

If the neonate survives infancy, hypotonia may improve and no longer be evident in childhood, although motor development may be delayed, and there may be cognitive delay and learning disabilities

If a mother has a recurrent genital HSV lesion at the time of delivery, the newborn should have: Observation only Treatment with Acyclovir until test results return Surface cultures and HSV-PCR done at 24 hours of age with observation

If there is a maternal history of HSV and this is not a first occurrence, the risk of transmission is low. If the infant is asymptomatic, surface cultures and a HSV-PCR on blood should be done at 24 hours of age, with the infant observed until the results are known

The immunoglobulins which are higher in colostrum as compared to later breast milk are IgA and: IgE IgM IgG

IgM and IgA are highest in colostrum. IgG remains constant throughout the first 180 days of milk production.

RCT

In an RCT, subjects are randomly assigned to either the control group or the experimental group (the group receiving the new drug). The purpose of randomization is to prevent allocation bias, which results when subjects in the experimental group are different from those in the control group (for example, patients with a mild disease vs those with severe disease). By assigning subjects randomly to the control and experimental groups, all factors - known and unknown, important and unimportant - should be equally represented in both groups.

Premature formula

Increased protein, vitamin and mineral content

Consideration that a positive blood culture result is the result of contamination is supported by: Growth of a single organism The same organism grows in multiple bottles Increased time to a positive result

Indicators of blood culture contamination are if there is increased time to positive culture (2-3 days or more), if there is growth of multiple organisms, and if different organisms grow in different bottles.

The severity of neonatal disease related to varicella exposure is greatest when mother has onset of chickenpox: 1-2 weeks prior to delivery 1 week prior to and 1 week following delivery 5 days prior to and 2 days following delivery

Infants whose mothers have the onset of the rash of varicella within 5 days prior to delivery and 2 days following delivery have a high risk of having the varicella virus transmitted to them due to lack of time for maternal antibody to develop and be transmitted to the fetus. In as many as 30% of infant who are untreated, the varicella may be disseminated and even fatal

Indomethacin does what?

Inhibits prostaglandin synthesis and matures the microvessels in the germinal matrix

Inserting the lumbar puncture needle between L4-L5 minimizes the development of: Cerebral tissue herniation Intraspinal epidermoid tumor Spinal cord and nerve damage

Inserting the needle between the L4-L5 interspace, where the spinal cord ends, minimizes the risk of spinal cord and nerve damage.

Jitteriness can be differentiated from seizures in the newborn in that jitteriness is: An isolated event that does not occur in response to stimuli Associated with movements that are of unequal amplitude Eliminated by flexing and holding the involved extremity

Jitteriness can be distinguished from seizure activity in that jitteriness is characterized by rapid alternating movements of equal amplitude in both directions and clonic movements seen during true seizures have a fast and slow component and are not as rapid. Noise, touch, or other environmental stimuli can elicit jitteriness, which can be stopped by flexing or holding the involved extremity. Seizures are generally not initiated by stimulation, nor can they be eliminated by flexing and holding. In addition, jitteriness is not associated with any subtle signs of seizure activity, such as abnormal eye movement.

Chromosomal karyotyping is done on a neonate with a cleft palate, micropenis and cryptorchidism. The results are 47 XXY. Theses findings are consistent with: Mixed gonadal dysgenesis Turner syndrome Klinefelter syndrome

Klinefelter syndrome is most often 47 XXY, although there are other variants. Infants with Klinefelter syndrome often have cleft palate, micropenis and cryptorchidism. They may also have hypospadias and inguinal hernia.

Maternal infection with listeria generally occurs as a result of which of the following? Eating soft cheeses Eating fresh fruit Drinking pasteurized milk

L. monocytogenes is acquired by susceptible adults (those with lower cellular immunity) through contaminated food. Common sources of Listeria include: soft cheeses, delicatessen meats, hot dogs, smoked seafood and pates

soy formula

Lactose free b. Indications i. galactosemia, hereditary lactase deficiency, parents wanting vegetarian diet

Which statement applies to late-onset sepsis? Use of proton pump inhibitors is a risk factor Listeria monocytogenes can be a causative organism It is acquired by vertical transmission

Late-onset sepsis is more frequently caused by coagulase-negative staphylococcus, staphylococcus aureus, gram negative organisms and candida. Proton pump inhibitor use is a risk factor for gram negative organisms in late onset sepsis. Late-onset sepsis is acquired from the environment and caregivers

A consideration when adding Levetiracetam to anti-seizure therapy is: Bioavailability is only 50% It may be used as solo therapy Weaning the dose, rather than abrupt discontinuation, is recommended

Levetiracetam is an adjunctive anti-seizure therapy, used in addition to other anti-seizure medications in infants >1 month of age. It has 100% bioavailability, but must be gradually weaned, rather than abruptly discontinuing the medication.

In contrast to Amphotericin B, Liposomal Amphotericin B preparations have: Increased toxicity Poorer CNS penetration A faster onset of action

Liposomal Amphotericin B preparations provide for better deliver to the reticuloendothelial system organs (liver, lungs and spleen), but penetrate the central nervous system less. It has a slower onset of action and reduced toxicity

The drug of choice for the treatment of Listeria monocytogenes infection is: Ampicillin Linezolid Ceftriaxone

Listeria monocytogenes infection should be treated with Ampicillin. In addition, an aminoglycoside or Cefotaxime should be added

Malpractice is professional misconduct that may be: Intentional only Intentional or unintentional Unintentional only

Malpractice is when the standards of ordinary nursing practiced by nurses of similar background in the same specialty has been violated, and it may be intentional or unintentional.

Maternal chorioamnionitis is a risk factor for neonatal: Cranial ultrasound abnormalities Omphalitis Osteomyelitis

Maternal chorioamnionitis increases risk for necrotizing enterocolitis, cranial ultrasound abnormalities and long-term neurologic impairment.

Maternal Parvovirus B-19 infection during pregnancy can result in which hematologic disorder in the newborn: Hyperbilirubinemia Leukemia Anemia

Maternal parvovirus B19 infection can result in severe anemia, and fetal hydrops. It can also cause severe thrombocytopenia.

Arthrogryposis

Means "curved joints" Multiple joint contractures (at least 2 or more) in multiple body areas Etiology is unknown Treatment to avoid fixed deformities

A type of evidence synthesis in which data from several previous studies are pooled is a/n: Integrative analysis Qualitative analysis Meta-analysis

Meta-analysis is by definition a large analysis of pre-existing data.

In a neonate receiving therapeutic hypothermia and phenobarbital, the phenobarbital level may exceed therapeutic levels with a normal dose primarily due to hypothermia's impact on: Calcium channel blockade Glomerular protein accumulation Cytochrome P450 metabolism

Metabolism of phenobarbital, morphine and vecuronium are slowed by the effects of temperature dependent hepatic cytochrome P450 metabolism, potentially leading to toxic levels

Presentation of hyperbilirubinemia secondary to hyperalimentation may present as soon as which length of time on total parenteral nutrition (TPN)? 1 week 2 weeks 4 weeks

Most cases of TPN associated cholestasis occur with prolonged use of TPN for 2 weeks or longer, with 90% of infants developing cholestasis within 13 weeks.

Achondroplasia

Most common skeletal dysplasia Autosomal dominant May be associated with advanced paternal age Monitor growth Baseline imaging- leg bowing, skull Monitor for obstructive apnea Megalocephaly- may develop hydrocephalus, Low nasal bridge, Long philtrum, Kyphosis, Shortened limbs, Flaring metaphyses, Squared off iliac wings, Deep sacrum

Neural tube defects are an example of: Multifactorial inheritance Mendelian inheritance Non-Mendelian inheritance

Most neural tube defects are multifactorial. While they cluster in families, folic acid levels are not only impacted by dietary intake, but also by enzymatic processes due to a genetic mutation

how to calculation anions in TPN

Na + K - phos = how much you have to work with chloride and acetate

The genetic transmission of Spinal Muscular Atrophy (SMA) is: Autosomal recessive Sex-linked Autosomal dominant

Neonatal SMA (Werdnig Hoffman disease or SMA Type I) is autosomal recessive.

Cafe au lait spots measuring 1.5cm or larger or more than three in number may indicate: Fibromalasia Neurofibromatosis Strurge-Weber Syndrome

Neurofibromatosis type 1 should be suspected in any infant with six or more cafe-au-lait macules of 0.5cm before puberty (1.5cm postpuberty)

White blood cells

Neutrophils: Primarily responsible for killing and digesting bacteria Eosinophils: Usually associated with allergic diseases Basophils: Inflammatory reactions, particularly those that cause allergic symptoms Lymphocytes: Part of our immune defense and act to recognize antigens, produce antibodies, and destroy cells that could cause damage Monocytes: The main scavenger cells of the immune system and regulate immunity against foreign substances.

Deformation

Normal development, but this process is acted upon by an extrinsic or intrinsic mechanical force 1. Club feet, Potter's facies, plagiocephaly

Which one of the following statements about the relationship between Apgar score and cerebral palsy is true? A newborn with an Apgar score of 3 at 15 minutes has a higher risk of cerebral palsy than death Improvement in Apgar score between 5 and 10 minutes decreases the cerebral palsy risk The majority of children who develop cerebral palsy had a normal Apgar score at 5 minutes of age

Of children who develop cerebral palsy, 73% have a 5 minute Apgar score of 7-10. A newborn with a low Apgar score at 1 minute, which improves by 10 minutes has only a 1% risk of developing cerebral palsy, and a newborn with a low Apgar score at 15 minutes has a greater risk of death that developing cerebral palsy.

Neonates are at an increased risk for otitis media include if they: Are nasotracheally intubated Have micrognathia Are female

Otitis media occurs more frequently in male than in female infants, in infants with cleft palate, and in infants with low birthweight who require prolonged nasotracheal intubation

A weak cry with a cyanotic episode may be indicative of: Phrenic nerve palsy Radial nerve palsy Brachial plexus palsy

Phrenic nerve palsy is caused by injury to the cervical nerve roots at C3 to C5. Risk factors include vaginal delivery of LGA infants, shoulder dystocia, breech delivery and other abnormal presentations, prolonged labor and difficult delivery. Because the diaphragm is paralyzed, infants with phrenic nerve injury have labored breathing, tachypnea, cyanosis, and can have apneic spells

In the diagnosis of otitis media in the infant: Culture of the throat and nasopharynx is beneficial Pneumatic otoscopy is recommended Tympanometry may be needed

Pneumatic otoscopy is recommended as the pliable ear canal walls can be more easily expanded

Which statement describes Sturge-Weber syndrome? Magnetic resonance imaging indicates decreased myelination of the spinal cord Port-wine stains noted in the upper eyelid or the forehead indicate the need for further evaluation Ophthalmologic findings upon evaluation include neonatal cataracts and scleral hemorrhage

Port wine nevi involving both eyelids, with bilateral distribution, and those that are unilateral but involve all three branches of the trigeminal nerve are associated with significantly higher incidence of eye and central nervous system abnormalities. They may indicate the presence of Sturge-Weber syndrome with underlying arteriovenous malformations

Elements that must be present for a prescription

Prescriber name, license number, and contact information • Prescriber DEA number, if applicable • Patient name and date of birth • Patient allergies • Name of medication • Indication of medication (e.g., atenolol for hypertension) • Medication strength (e.g., 25 mg, 500 mg/mL) • Dose of medication and frequency (e.g., 12.5 mg once daily) • Number of tablets/capsules to dispense • Number of refills

Vernix

Production occurs at time of epidermal cornification Develops in the last trimester from the hair follicles Lacking at gest less than 28wks Facilitates SC formation Protective from water exposure Retention at birth promotes increased hydration and decreased pH

Which of the following categorize and describe data according to patterns, themes, and categories of response at a nominal level? Qualitative studies Quantitative studies Developmental studies

Qualitative studies categorize and describe data according to patterns, themes, and categories of response at a nominal or naming level.

The shunt associated with pulmonary hypertension is: Right to left Left to right Aorta to pulmonary artery

Right to left

Vocal cord paralysis

Second most common cause of stridor ii. Stridor is inspirational or biphasic iii. Often symptomatic first week of life, not always congenital 1. Unilateral: damage to vagus or recurrent nerve via traumatic delivery, CT surgery (PDA ligation, repair of tracheal-esophageal fistula); cardiac atrial enlargement or anomalous great vessels. 2. Bilateral: neurologic abnormalities. v. History of coughing, choking, or cyanosis during feedings vi. CXR or Echocardiogram to examine cardiac size; swallow study, US vii. CT to map course of vagus nerve or evaluate for neurologic abnormalities viii. Laryngoscopy may be difficult due to laryngomalacia ix. Treatment: Unilateral-prevent aspiration through dietary changes and will usually resolve by 6-12 months; bilateral-often requires tracheostomy

The differential diagnosis for seizures in the neonate includes: Hypoglycemia, infection and hypernatremia Hyperglycemia, hypercalcemia and hyponatremia Hypoglycemia, infection and drug withdrawal

Seizures are a clinical manifestation associated with various underlying pathologic process including hypoxia, ischemia, hypoglycemia, hypocalcemia, intracranial hemorrhage, infection (meningitis, congenital viral infections, viral encephalopathy), congenital anomalies of the CNS, and other metabolic disturbances, such as alkalosis, hypomagnesemia, hypernatremia and hyponatremia, drug withdrawal and amino and organic acid metabolism, kernicterus, hyperviscosity, and local anesthetic intoxication.

Bilirubin can be displaced from albumin by: Indomethacin Phenytoin Fentanyl

Several medications can displace bilirubin from albumin including: Ampicillin, Indomethacin salicylates and sulfa drugs. Phenytoin is displaced from albumin by bilirubin

SMOF lipids

Soy oil, Medium chain triglycerides, Olive oil, Fish Oil b. Contain omega-3 and medium chain triglycerides c. Closely resembles the omega-6:omega-3 ratio in breast milk d. Higher concentration on vitamin E e. Some evidence suggests that SMOF use may decrease the incidence and severity of parenteral nutrition associated liver disease

Pharmacogenetics

Study of the role of genetic factors in drug disposition, response, and toxicity Relates variability in human genes and drug responses in individual patients

The neurologic findings with Sturge-Weber syndrome are due to: Cortical tubules Spongiform changes in the white matter Vascular proliferation with cerebral calcifications

Sturge-Weber syndrome is a neurocutaneous syndrome which causes leptomeningeal vascular proliferation and cerebral cortical calcifications

Patients with HIE undergoing cooling are at risk for

Subcutaneous fat necrosis (SCFN) of the newborn

Which of these hemorrhages is most often secondary to birth trauma due to use of forceps or vacuum extraction? Intraventricular Subarachnoid Subgaleal

Subgaleal hemorrhage is typically related to traumatic events surrounding the delivery and birth and use of forceps or vacuum extraction. Caused by pressure and tractions on the emissary veins, it can be a source of significant blood loss, and can be fatal.

According the American Academy of Pediatrics (AAP), which infant should receive RSV prophylaxis with palivizumab during RSV season: An infant born at 31 weeks gestation with no other risk factors An infant with a small ventricular septal defect An infant diagnosed with congenital myotonic dystrophy

The AAP RSV prophylaxis guidelines recommend that the following infants receive prophylaxis monthly during RSV season: 1. Infants born at 29 weeks gestation or less, 2. Infants with chronic lung disease (defined as born at less than 32 weeks 0 days gestation with a need for >21% oxygen at 28 days of age) 3. Infants with hemodynamically significant congenital heart disease (a small VSD does not fit this criteria), and 4. Infants with congenital pulmonary or neuromuscular conditions that compromise handling of respiratory secretions and 5. Infants who will be profoundly immunocompromised during RSV season. American Academy of Pediatrics

The one way of analysis of variance (ANOVA) tests the statistical significance of: Two values obtained from paired subjects Three or more group means Different proportions

The ANOVA tests the statistical significance of three or more group means.

A Rapid Plasma Reagin (RPR test) is considered positive for infant infection if it is: Five-fold higher than the mother's Two fold higher than the mother's Four-fold higher than the mother's

The RPR is a non-specific nontreponemal antibody test. A titer that is 2 dilutions (four-fold) higher in the infant than in the mother signifies probable infection in the infant.

Newborns who have sustained cerebral hypoxia-ischemia present with: Variable clinical symptoms reflective of the timing, severity, and duration of the insult to the brain Profound symptoms with predictable improvement Neurologic deficits that will not improve over time

The clinical course after a hypoxic-ischemic event in labor is unpredictable, so problems should be anticipated and the infant appropriately observed by trained staff in a clinical environment. Hypoxic-ischemic insult affects the whole organism and any organ may be compromised.

Which clinical presentation of intraventricular hemorrhage in a premature infant is most common? Asymptomatic Catastrophic Saltatory

The clinical presentation of IVH is diverse, with three classic presentations. The most common is asymptomatic, with no clinical signs. The second is saltatory, with a gradual onset in change of the infant's movements. Catastrophic deterioration is reflected by a sudden change in the infant's clinical condition.

A cystic dilation of the fourth ventricle, complete or partial agenesis of the cerebellar vermis, and hydrocephalus describes: Dandy-Walker Malformation Holoprosencephaly Hydranencephaly

The components of the Dandy- Walker malformation are cystic dilation of the fourth ventricle, partial or complete agenesis of the corpus callosum, and enlargement of the posterior fossa with a high attachment of the tentorium cerebelli. The Dandy-Walker malformation is frequently associated with hydrocephalus, which may not be present at birth but often develops in the first year of life.

Which complication would be associated with the postoperative Ventral Septal repair period? Cardiac arrhythmias Pulmonary hypertension Right ventricular failure

The correct answer is "Cardiac arrhythmias". VSD closure repair complications can include right bundle branch block, third-degree heart block, aortic insufficiency, tricuspid insufficiency.

The imaging study called a HIDA scan is done to assess for which of the following diagnoses? Cholestasis Eagle-Barrett syndrome Gastroesophageal reflux

The correct answer is "Cholestasis". The HIDA scan, or hepatobiliary scintigraphy scan, evaluates bile flow and for possible biliary obstruction.

Which of the following cardiac defects is ductal-dependent requiring a prostaglandin E1 infusion? Ebstein's anomaly Endocardial cushion defect Aortopulmonary window

The correct answer is "Ebstein's anomaly". The anterior leaflet is redundant and could obstruct the right ventricular outflow tract, resulting in decreased blood flow and functional or anatomic pulmonary atresia. Maintaining ductal patency with prostaglandin E1 can be useful in the short term.

When every other atrial beat is conducted, it is called: 3:1 AV conduction 1:2 AV conduction 2:1 AV conduction

The correct answer is - " 2:1 AV conduction." The neonatal AV node cannot conduct atrial rates of 400 to 500 beats per minute in a 1:1 relationship, and there is some degree of AV block. If every other atrial beat is conducted, there is a 2:1 AV conduction (second degree block), if every third beat is conducted, there is 3:1 conduction (third degree block).

Polyhydramnios may indicate: No relativity to gastrointestinal defect A high gastrointestinal defect A low gastrointestinal defect

The correct answer is - "A high gastrointestinal defect." Polyhydramnios is idiopathic in 35-65% of cases, with advanced sonographic capability, the etiology can be detected in up to 90% of the cases of moderate to severe polyhydramnios. The major pathway of amniotic fluid removal is fetal swallowing, it follows that congenital anomalies that interfere with swallowing or intestinal absorption are the most common causes of polyhydramnios

Which of the following is a cause of fetal bradycardia? Fetal hypotension following complete cord occlusion which causes a baroreceptor reflex A placental abruption or uterine rupture effecting the fetal circulation Use of beta-sympathomimetic agents given to correct maternal hypotension following induction of epidural anesthesia

The correct answer is - "A placental abruption or uterine rupture effecting the fetal circulation." An acute severe bradycardia may be the response to an adverse effect on the fetal circulation, which would occur following a placental abruption or uterine rupture. Acute hypoxemia stimulates a chemoreceptor reflex, which causes bradycardia. An acute bradycardia could be the result of fetal hypertension following complete cord occlusion which causes a baroreceptor reflex, and stimulates a vagal response and a fall in FHR.

The finding of premature ventricular beats in a neonatal electrocardiogram (ECG) should lead to which of the following: Immediate administration of adenosine Immediate cardioversion Assessment for structural or functional cardiac disease

The correct answer is - "Assessment for structural or functional cardiac disease." When premature ventricular beats are identified in a newborn infant, a careful search for structural or functional cardiac disease is required, together with an evaluation to rule out electrolyte disorders.

Electrocardiogram (ECG) findings of "irregularly irregular, bizarre, chaotic, or difficult-to-see P waves" are consistent with which of the following neonatal arrhythmias? Normal sinus rhythm Ventricular tachycardia Atrial fibrillation

The correct answer is - "Atrial fibrillation." The characteristic ECG findings are irregularly irregular; no two R-R intervals exactly the same; P waves difficult to see or bizarre and chaotic

Which of the following may produce heart rates as high as 300-600 beats per minute in infants? Atrial flutter Ventricular tachycardia Ventricular fibrillation

The correct answer is - "Atrial flutter." Atrial flutter results from reentry involving a circuit in a large part of atrial muscle. Atrial flutter rates are as high as 400 to 500 beats per minute in newborn infants, in contrast with older children and adults, who normally have atrial rates of approximately 300 beats per minute.

Portal hypertension and cirrhosis of the liver are complications of: Biliary atresia Hepatitis Alpha one (a1) antitrypsin deficiency

The correct answer is - "Biliary atresia." Portal hypertension is a major complication of biliary atresia. The reverse blood flow results in enlargement of esophageal, umbilical, and rectal veins, which is manifested as splenomegaly, hemorrhoids, enlarged abdominal, ascites, and blood in the stools. Additional complications include decreased clotting ability, anemia and ineffective metabolism of nutrients. End-stage liver disease may lead to rupture of veins in the esophagus and stomach or hepatic coma with eventual death from liver failure.

Which of the following cardiac defects is most commonly associated with Turners syndrome? Coarctation of the aorta Endocardial cushion defect Pulmonary valve stenosis

The correct answer is - "Coarctation of the aorta." Turner's syndrome is associated with aortic stenosis, coarctation of the aorta and hypoplastic left heart syndrome

The pH of gastric contents in term newborn infants compared to premature infant is: Decreased The same Increased

The correct answer is - "Decreased." The gastric luminal pH of full-term newborns is neutral, but it is as low as 3.5 within a few hours. By 48 hours, the pH is 1.0 to 3.0. Premature infants have a prolonged period of alkalinity, often many days, that is related to degree of prematurity

Which of the following best describes the action of Adenosine? Has negative inotropic effects Depresses the AV node automaticity and SA node conduction Depresses the sinus node automaticity and AV node conduction

The correct answer is - "Depresses the sinus node automaticity and AV node conduction." Adenosine slows the sinus rate and produces transient AV block.

The layer of the skin which contains fibrous and elastic tissues, sweat glands, sebaceous glands, hair shafts, and many nerves and blood vessels is the: Dermis Epidermis Subcutaneous layer

The correct answer is - "Dermis." The skin consists of three anatomically distinct layers: the epidermis, the dermis, and the subcutaneous tissue. The outermost layer of the epidermis is the stratum corneum, the protective outer layer. It is mainly composed of dead cells that prevent chemicals and microorganisms from entering the body, and prevents insensible water loss when mature. The bottom layer of the epidermis, the basal layer, constantly replaces these cells. The dermis lies directly under the epidermis. It is a closely woven layer of collagen and is 2 to 4 mm thick at birth. Many nerves and a rich supply of blood vessels, fibrous and elastic tissues, sebaceous gland, hair shafts and cutaneous nerves are found in the dermis. The primary component of the subcutaneous layer is fatty connective tissue. This subcutaneous fat acts as a heat insulator, a shock absorber, and a calorie reserve.

The "double bubble" on an abdominal x-ray is a diagnostic sign for: Jejunoileal atresia Pyloric stenosis Duodenal atresia

The correct answer is - "Duodenal atresia." On a frontal chest and abdominal x-ray, gaseous distention of the stomach and duodenal bulb are the classic double-bubble seen in duodenal atresia

The triad of "excessive" drooling, cyanosis and respiratory distress, especially if associated with a maternal history of polyhydramnios, suggests which of the following? Gastroesophageal reflux (GER) Biliary atresia Esophageal atresia

The correct answer is - "Esophageal atresia." Most cases of EA/TEF are not diagnosed prenatally however, a combination of maternal polyhydramnios and a small or absent stomach has both sensitivity and a positive predictive value for EA/TEF of approximately 40% to 60%. The finding of a dilated upper pouch in the neck by high resolution ultrasonography or magnetic resonance imaging can increase the diagnostic accuracy in this selected group of patients to nearly 100%. The infant with esophageal will not be able to swallow secretions and will appear to drool excessively. As the upper pouch fills, tracheal compression and ante grade aspiration may occur, leading to significant coughing, respiratory distress, and cyanosis. In patients with a distal fistula, the stomach may dilate with air, leading to the reflux of gastric secretions back into the lungs, resulting in significant and progressive respiratory distress due to reactive bronchoconstriction and chemical pneumonitis

Vitamin K availability impacts which clotting factors? II, VII, IX, X II, VII, X, XII II, V, VII, X

The correct answer is - "II, VII, IX, X." Vitamin K is required for activation of clotting factors II, VII, IX and X and of protein C and protein S. There is inadequate placental transfer, and poor hepatic storage of Vitamin K, in addition to poor dietary intake in the first few days of life. In the absence of Vitamin K, the factors (coagulation proteins), are dysfunctional.

Which of the following is true of fat digestion in the newborn? In term infants, pancreatic lipase activity is diminished compared to the adult Bile acid secretion is increased in term infants compared to adults Clinically significant steatorrhea is a common and physiologic finding related to the term infant's immaturity compared to the adult

The correct answer is - "In term infants, pancreatic lipase activity is diminished compared to the adult." Fat digestion in the adult relies on pancreatic lipase to break down triglycerides and bile acids to emulsify fat droplets before and during lipolysis. These processes are decreased in term and especially preterm infants. Lipase activity at term is 10% to 20% of that seen in older children, partly due to minimal responsiveness by pancreatic acinar cells to secretin and cholescystokinin during the first month. Term infants fail to absorb 10% to 15% and preterm infants 30% or more of ingested fats. Lipase activity increases up to four-fold during the first week post-birth in healthy term infants. Bile acid synthesis and pool size are one half of adult values in term infants, and one sixth of adult and one third of term infant values in preterm infants.

Which of the following is an adverse effect of high-calorie feedings? Lactobezoars Hirschsprung disease Gastroesophageal reflux

The correct answer is - "Lactobezoars." Lactobezoars are a mixture of undigested milk curds and mucus that form a compact mass usually in the stomach. They are an adverse effect of high-caloric feedings. Lactobezoars are firm balls of fat that form in the infant's intestinal tract about 3 to 12 days after enteral feedings have started.

Which of the following is true of infants with gastroschisis? May arise as the result of incomplete lateral infolding of the embryonic disk Routine premature delivery of infants with gastroschisis is recommended C-section delivery has been proven to improve outcome

The correct answer is - "May arise as the result of incomplete lateral infolding of the embryonic disk. Gastroschisis is thought to arise as the result of incomplete lateral infolding of the embryonic disk. As a result of this primary failure, the abdominal wall is incompletely formed, allowing herniation of the gut. Cesarean section is often chosen for deliver to avoid injury to exposed bowel, but studies have shown no difference in mortality or morbidity for infants born vaginally or by C-Section. Some clinicians have proposed premature delivery when there is evidence of bowel compromise, but routine preterm delivery is not recommended because of the inherent risk factors of prematurity and the reversibility of most intestinal damage.

Which of the following is consistent with gastroschisis? Covered with amnion membrane High incidence of associated defects Morbidity is mostly related to in utero injury to the bowel

The correct answer is - "Morbidity is mostly related to in utero injury to the bowel." Morbidity in infants with gastroschisis, as opposed to those with omphalocele, is almost entirely related to intestinal dysfunction caused by in utero injury to the eviscerated bowel

Which of the following is true of fetal tachycardia? Most commonly caused by maternal infection, chorioamnionitis May be the result of a fetal hypertension following complete cord occlusion Can be seen following sublethal hyperoxic events

The correct answer is - "Most commonly caused by maternal infection, chorioamnionitis." The most common etiology of sustained tachycardia in the term fetus is maternal infection, specifically chorioamnionitis. Fetal tachycardia can also be seen following sublethal hypoxic events that cause catecholamine release and sympathetic stimulation. Other sources of fetal tachycardia are the use of beta-sympathomimetic agents to correct maternal hypotension following induction of epidural anesthesia, fetal anemia (Rh isoimmunization), acute fetal blood loss (placental abruption), or abnormal fetal conduction system (fetal arrhythmia).

Paddle placement for neonatal cardioversion includes which of the following: Sternal border mid chest and to the right of the right nipple One pad on the right upper sternum and one pad below the left nipple toward the axilla Anterior posterior position where one paddle is placed on the back between the scapulae, second paddle is placed on the right upper sternum

The correct answer is - "One pad on the right upper sternum and one pad below the left nipple toward the axilla." There are two different positions for pad placement. 1. Anterior -lateral positioning - The anterior pad is placed to the right of the upper sternum, and the posterior pad placed below the left nipple toward the axilla. 2. Anterior-posterior positioning - The anterior pad is placed on the midsternal border and the posterior pad is placed between the scapulae

Rarely, an omphalocele defect is cephalad to the umbilicus producing a complex deformity referred to as: Pentalogy of Cantrell Giant Omphalocele Cloacal exstrophy

The correct answer is - "Pentalogy of Cantrell." Rarely, the omphalocele defect is cephalad to the umbilicus, producing a complex deformity referred to as pentalogy of Cantrell: sternal, diaphragmatic, and pericardial defects, upper abdominal omphalocele, and ectopic cordis. When centered below the umbilicus, bladder or cloacal exstrophy may occur.

Which of the following is a radiographic finding of necrotizing enterocolitis (NEC)? "Pigtail" or 'funnel" Pneumatosis intestinalis "Double-bubble"

The correct answer is - "Pneumatosis intestinalis." Radiologic evidence includes the presence of pneumotosis intestinalis where gas dissects beneath the serosal and submucosal layers of the intestine. If the gas ruptures into the mesenteric vascular bed, it can distribute through the systemic vessels into the venous system of the liver forming portal venous gas seen as branching vessels in the liver. Finally, if the intestines rupture, a pneumoperitonium or gas outside the bowel wall can be seen.

Sinus bradycardia is a common occurrence in the newborn period due to which of the following: Complete sympathetic cardiac innervation Predominance of vagal innervation Primary parasympathetic control

The correct answer is - "Predominance of vagal innervation." Newborn infants are thought to have incomplete sympathetic cardiac innervation and are predominantly vagally innervated. As a result of this autonomic imbalance, episodic sinus bradycardia is fairly common

Which of the following contributes to the high levels of unconjugated bilirubin in the newborn intestine? Increased bacterial flora Decreased hydrolysis of bilirubin glucuronide Presence of meconium

The correct answer is - "Presence of meconium." Neonates have relatively high concentrations of unconjugated bilirubin in the intestine, derived from increased bilirubin production, exaggerated hydrolysis of bilirubin glucuronide, and high concentrations of bilirubin found in meconium. In the newborn, the relative lack of bacterial flora to reduce bilirubin to urobilinogen further increases the intestinal bilirubin pool in comparison with that of the older child and adult.

The appropriate medication to use for ductal-dependent lesions includes which of the following? Prostaglandin E Indomethacin Lasix

The correct answer is - "Prostaglandin E." Neonates with severe cyanosis are stabilized by an infusion of prostaglandin E1 to dilate the ductus arteriosus and increase pulmonary blood flow.

Which of the following describes Long QT Syndrome: Abnormal P waves QT intervals of > 0.44 seconds Irregular, rapid rate

The correct answer is - "QT intervals of > 0.44 seconds". The QT interval exceeds the normal range, the P and QRS waveforms will be normal, there will be a normal rate and the T wave may be notched, biphasic or inverted with Prolonged or Long QT Syndrome (LQTS).

On an electrocardiogram (EKG) strip, the t wave represents: Depolarization of the ventricles Repolarization of the ventricles Repolarization of the atria

The correct answer is - "Repolarization of the ventricles." The ST segment is followed by a larger rounded wave known as the T wave. The T wave represents repolarization of the ventricles

Which of the following may be a sign of pneumoperitoneum: Scaphoid Abdomen Respiratory distress Hypertension

The correct answer is - "Respiratory Distress. " Depending on the cause and severity, pneumoperitoneum can present with or without associated abdominal findings. Because pneumoperitoneum can occur as a result of pneumothorax, pneumomediastinum, and pulmonary interstitial air, infants can present with signs of respiratory distress.

Match the condition, Tetralogy of Fallot, with the one of characteristics listed below. Severe cyanosis caused by separate circulations and poor mixing Severe cyanosis caused by restricted pulmonary blood flow Mild cyanosis caused by complete mixing with normal or increased pulmonary blood flow

The correct answer is - "Severe cyanosis caused by restricted pulmonary blood flow." One of the characteristics of Tetrology of Fallot is severe cyanosis caused by restricted pulmonary blood flow. In TOF there is an association of infundibular (subvalvar) and valvar pulmonary stenosis, a large ventricular septal defect, a large aorta overriding the ventricular septum, and right ventricular hypertrophy. A variable degree of pulmonary artery hypoplasia, an atrial septal defect or foramen ovale, patent ductus arteriosus, or systemic-to-pulmonary collateral vessels are often present. In most severe form of TOF, atresia of the pulmonary valve or the entire right ventricular outflow tract (OFT) and main pulmonary artery occurs.

Match the condition, Transposition of the Great Arteries, with one of the characteristics listed. Severe cyanosis caused by restricted pulmonary blood flow Mild cyanosis caused by complete mixing with normal or increased pulmonary blood flow Severe cyanosis caused by separate circulations and poor mixing

The correct answer is - "Severe cyanosis caused by separate circulations and poor mixing." In contrast to the blood flow pattern in the normal heart, in which the pulmonary and systemic circulations are in series, in transposition of the great arteries the pulmonary and systemic circulations are in parallel. Then the venous blood returning to the right atrium flows through the tricuspid valve into the right ventricle and is ejected into the aorta, supplying the body with poorly oxygenated blood, which then returns to the right atrium to repeat the cycle. Fully oxygenated blood from the lungs enters the left atrium, flows through the mitral valve into the left ventricle, and is ejected into the pulmonary artery to perfuse the lungs and return to the left atrium without delivering oxygen to the body. In the absence of shunting across connections between systemic and pulmonary circulations, this results in profound, lethal systemic hypoxia.

Toxic cardiac effects of digoxin use include which of the following: Sinus bradycardia Deceleration of junctional rhythm Sinus tachycardia

The correct answer is - "Sinus bradycardia." Children are more likely to experience cardiac arrhythmias as a sign of excessive dosing. The most common are conduction disturbances or tachyarrhythmia and junctional tachycardia. Ventricular tachyarrhythmias are less common. In infants, sinus bradycardia may be a sign of digoxin toxicity. Acceleration of junctional rhythm may also be seen

Intestinal duplications most often involve the: Small intestine Colon Stomach

The correct answer is - "Small intestine." Duplications of the small or large intestine are more common than those of the stomach and duodenum. They are usually cystic but may be tubular and extend for a variable distance. Cystic duplications may cause obstruction or volvulus, and they may present with a palpable mass.

Which of the following spontaneously resolves within the neonatal period and is frequently seen in infants of diabetic mothers? Small left colon syndrome Meconium ileus Hirschsprung disease

The correct answer is - "Small left colon syndrome." Approximately 40% of those with small left colon syndrome are the infants of mothers with diabetes. Management is generally conservative. The diagnostic barium enema is generally curative. The condition spontaneously resolves within the neonatal period with no subsequent stooling problems encountered. Late intermittent obstruction with or without cecal perforation is reported rarely

The following electrocardiogram (ECG) findings are consistent with what neonatal arrhythmia? - Consistent rate that is not variable with environmental factors - Rates often greater than 240 beats per minute Supraventricular tachycardia Ventricular fibrillation Atrial flutter

The correct answer is - "Supraventricular tachycardia." Criteria for (SVT) supraventricular tachycardia include (1) persistent ventricular rate over 200 beats/min, (2) a fixed and regular R-R interval, and (3) little variability in heart rate with various activities (e.g. crying, feeding, apnea).

The effects of congestive heart failure include which of the following: Bradycardia Apnea Tachypnea

The correct answer is - "Tachypnea." Common signs of CHF include: tachypnea, tachycardia, arrhythmias, cardiomegaly, hepatomegaly, pulmonary fine rales and coarse rales (rhonchi), fatigue or difficulty with feeding leading to failure to thrive, peripheral edema, diaphoresis, gallop rhythm, altered pulses, mottling.

Mothers who have diabetes mellitus during cardiogenesis have a twofold to fourfold risk for having babies with congenital heart disease. Which of the following congenital heart diseases would be most common with this condition? Tetralogy of Fallot Atrial septal defect Double-outlet left ventricle

The correct answer is - "Tetralogy of Fallot." The congenital heart diseases most commonly seen with maternal diabetes mellitus during cardiogenesis are ventricular septal defect, transposition of the great arteries, Tetralogy of Fallot, and double-outlet right ventricle

Mothers who have diabetes mellitus during cardiogenesis have a twofold to fourfold risk for having babies with congenital heart disease. Which of the following congenital heart diseases would be most common with this condition? Tetralogy of Fallot Atrial septal defect Double-outlet left ventricle

The correct answer is - "Tetralogy of Fallot." The congenital heart diseases most commonly seen with maternal diabetes mellitus during cardiogenesis are ventricular septal defect, transposition of the great arteries, Tetralogy of Fallot, and double-outlet right ventricle.

Which of the following is true of enterohepatic absorption of bilirubin? Conjugated bilirubin is readily absorbed from the intestine B-glucuronidase is present in low concentrations in the newborn's enteric mucosa The mildly alkaline conditions of the duodenum and jejunum support conjugated bilirubin to be hydrolyzed to unconjugated bilirubin

The correct answer is - "The mildly alkaline conditions of the duodenum and jejunum support conjugated bilirubin to be hydrolyzed to unconjugated bilirubin." Conjugated bilirubin is not absorbed from the intestine, whereas unconjugated bilirubin is readily and rapidly absorbed, participating in the enterohepatic circulation along with bile salts, phospholipids, cholesterol, certain hormones (thyroxine), and other endogenous and exogenous substances. The monoglucuronides and diglucuronides of bilirubin are relatively unstable conjugates that are readily hydrolyzed to unconjugated bilirubin both nonenzymatically, under the influence of mild alkaline conditions as in the duodenum and jejunum, and enzymatically, by enteric mucosal enzyme B-glucuronidase, which is present in both the term and the premature neonate in very high concentrations. Unconjugated bilirubin then may be reabsorbed across the intestinal mucosa to return to the liver via the portal circulation.

The essential difference between gastroschisis and omphalocele is: The site of the defect is at the umbilicus in an omphalocele and to the side of the normally placed umbilicus in gastroschisis The presence or absence of a sac The presence or absence of prematurity

The correct answer is - "The site of the defect is at the umbilicus in an omphalocele and to the side of the normally placed umbilicus in gastroschisis." In the case of omphalocele, a central abdominal wall defect of variable size is covered by a domelike mesenchymal membrane composed of amnion. The umbilical cord connects to the central portion of this membrane. The underlying abdominal organs are protected from exposure to amniotic fluid. In gastroschesis, the defect is usually smaller and located to the right of the umbilical attachment. There is no membranous covering. The abdominal contents, therefore are eviscerated and suspended in amniotic fluid during gestation.

The importance of distinguishing between gastroschisis and omphalocele is: There is increased frequency of associated anomalies with omphalocele Treatment in the delivery room varies significantly There is increased frequency of associated anomalies with gastroschisis

The correct answer is - "There is increased frequency of associated anomalies with omphalocele." Omphalocele occurs in roughly 1 of 3000 to 6000 live births, with a male predominance. Mothers tend to be younger (93% < 29 years old). Multiple and often life-threatening syndromes and anomalies occur with an unusually high frequency (30% to 70%) and include trisomy 13, trisomy 18, Beckwith-Wiedemann syndrome, pentalogy syndrome, congenital heart defects, diaphragmatic and upper midline defects, malrotation, intestinal atresia, and genitourinary anomalies. Additionally, 30% to 33% of affected infants are premature, and approximately 19% are small for gestational age. Of the infants with central omphalocele, about one quarter will have cardiac anomalies, especially Cantrell pentalogy (cleft sternum, anterior midline diaphragmatic defects, a pericardial defect, congenital cardiac abnormalities which may include ectopia cordis, and upper abdominal omphalocele). Lower omphaloceles are associated in some cases with Beckwith-Wiedemann syndrome (omphalocele, macroglossia, macrosomia, and hypoglycemia).

Which of the following is true of pregnant mothers with systemic lupus erythematosus (SLE)? There is transplacental passage of IgG antibody and complement deposition in the fetal myocardium Those with anti-Ro or anti-La have an approximate 50% risk of fetal heart block Risk of fetal heart block is related to severity of maternal disease

The correct answer is - "There is transplacental passage of IgG antibody and complement deposition in the fetal myocardium." Mothers with anti-Ro or anti-La antibodies because of lupus erythematosus have a small (approximately 5%) but important risk of fetal heart block. There is a transplacental passage of immunoglobulin G (IgG) antibody, with antibody and complement deposition in the fetal myocardium. The risk is not related to the severity of the maternal disease, and the diagnosis of the maternal collagen vascular disease could be prompted by the observation of fetal conduction abnormalities.

On an electrocardiogram (EKG) strip, the QRS complex represents: Ventricular repolarization Ventricular depolarization/contraction Atrial depolarization

The correct answer is - "Ventricular depolarization/contraction." The Q wave is the first downward stroke of the QRS complex. It may not always be present. It is followed by the upward R wave and then by the downward S wave. The entire QRS complex represents ventricular depolarization/contraction.

The most common way to recognize ventricular tachycardia in the newborn is to evaluate for which of the following: Prolonged QT interval Narrow QRS complexes Wide QRS complexes

The correct answer is - "Wide QRS complexes." Ventricular tachycardia is recognized from the wide QRS complexes of greater than 80 ms. This is a dangerous rhythm because rapid ventricular contractions prevent the ventricles from completely filling and emptying with each contraction.

Classification of the four major subtypes of epidermolysis bullosa is based on the: Region of the skin impacted Genetic mutation Amount of blistering

The correct answer is - Region of the skin impacted. The four major subtypes of epidermolysis bullosa are: Simplex (Epidermolytic), Junctional (Lucidolytic), Dystrophic (Dermolytic) and Mixed. The classification is based on the region of the skin in which the defective or missing protein occurs. In addition, the classification reflects the depth of blistering and the specific missing protein(s).

Diagnosis of esophageal atresia is confirmed by: Attempts to pass a radiopaque catheter from the nares through the esophagus into the stomach Contrast studies Ultrasonography

The correct answer is -" Attempts to pass a radiopaque catheter from the nares through the esophagus into the stomach." Diagnosis of esophageal atresia is confirmed by attempting to pass a radiopaque catheter from the nares through the esophagus into the stomach.

Congestive heart failure can best be described as: Hypotension Increased stroke volume Blood supply to the body is insufficient to meet the metabolic demands of the organ systems

The correct answer is -" Blood supply to the body is insufficient to meet the metabolic demands of the organ systems." CHF is a set of clinical signs and symptoms that reflect the heart's inability to deliver adequate oxygen to meet the metabolic requirements of the body.

The most recommended and safest treatment for gastroesophageal reflux disease (GERD) is: Wait out symptoms until infant is older gestation and more mature Perform surgical fundoplication Administer oral histamine 2 receptor agonist medication

The correct answer is -" Wait out symptoms until infant is older gestation and more mature". The majority of GER in infants in non-acidic in origin. Increasing gastric pH raises safety concerns, particularly related to Necrotizing enterocolitis. Fundoplication should be the last resort option due potential surgical and post-operative complications. Conservative expectant management is the mainstay of GERD in infants.

In patients with esophageal atresia and tracheoesophageal fistula, the most common lesion consists of: A blind upper pouch with a distal fistula Tracheoesophageal fistula without atresia Esophageal atresia of the upper and lower pouches without fistula

The correct answer is -"A blind upper pouch with a distal fistula." The most common variant is the combination of a proximal esophageal atresia and a distal tracheoesophageal fistula. The proximal atresia/distal fistula variant occurs in about 85% of affected patients

Match the condition, ventricular septal defect, with one of the characteristics listed below. Mild cyanosis caused by complete mixing with normal or increased pulmonary blood flow Acyanotic with no or mild respiratory distress Systemic hypoperfusion and congestive heart failure with mild or no cyanosis

The correct answer is -"Acyanotic with no or mild respiratory distress." In the neonatal period, most infants with uncomplicated left-to-right shunts, including ventricular septal defect and patent ductus arteriosus, are in the group of acyanosis with no or mild respiratory distress

Which of the following is a common adverse effect of Alprostadil (Prostaglandin E1)? Fever Seizures Diarrhea

The correct answer is -"Fever." Common adverse effects of Prostaglandin E1 include: apnea, hypotension, fever, leukocytosis, cutaneous flushing, and bradycardia. Hypocalcemia, hypoglycemia, hypokalemia, hyperkalemia are metabolic adverse reactions seen with Prostaglandin E1. GI reactions include diarrhea and gastric-outlet obstruction. Other reactions include inhibition of platelet aggregation and cortical proliferation of long bones seen with long term infusions.

Intrahepatic portal venous air is diagnostic of which of the following diseases? Necrotizing enterocolitis (NEC) Malrotation with volvulus Hirschsprung disease

The correct answer is -"Necrotizing enterocolitis (NEC)." If gas ruptures into the mesenteric vascular bed, it can distribute through the systemic vessels into the venous system of the liver forming portal venous gas seen as branching vessels in the liver.

Gastroesophageal reflux disease (GERD), short gut syndrome, and failure-to-thrive are sequelae associated with: Post-maturity Asphyxia Prematurity

The correct answer is -"Prematurity." NEC is a disorder seen primarily in preterm infants. Infants with reduction of their small intestine, which usually results from surgical resection because of a congenital anomaly or NEC, are at very high risk of nutritional and growth problems due to reduction in intestinal absorptive surface area. Infants are more susceptible to regurgitation, vomiting and gastroesophageal reflux (GER) due to anatomic and functional immaturity of their GI system. These limitations and the frequency of regurgitation are more marked in preterm infants. GERD is also seen more frequently in infants, particularly in preterm and ill neonates.

Which of the following are signs of pathologic gastroesophageal reflux (GER), as opposed to physiologic GER? Respiratory symptoms, failure to thrive, esophagitis Excessive irritability, apnea, weight gain Mild irritability, apnea, feeding aversion

The correct answer is -"Respiratory symptoms, failure to thrive, esophagitis." Surgical treatment for GER is reserved for high-risk infants who have complications for GER and for those who are refractory to medical therapy. Complications of GER include failure to thrive, aspiration pneumonia, reactive airway disease, and esophagitis with bleeding, stricture, ulceration, or Barrett's metaplasia.

Which of the following is considered a normal newborn electrocardiogram (EKG) finding? Left axis deviation Left ventricular hypertrophy Right ventricular predominance

The correct answer is -"Right ventricular predominance." Use of the electrocardiogram in the diagnosis of congenital heart disease is largely applicable to ventricular hypertrophy. Normal newborn infants have a relative predominance of the right ventricle over the left ventricle. This predominance shifts to left ventricular predominance in the first year of life because of the fall in pulmonary artery pressure and the consequent fall in right ventricular systolic pressure. All electrocardiographic interpretations in children must be made relative to the normal right or left ventricular predominance existing at that particular age.

Which of the following is a true statement about necrotizing enterocolitis (NEC)? The development of necrotizing enterocolitis (NEC) occurs due to mesenteric ischemia, bacteria, and feeding Necrotizing enterocolitis (NEC) routinely affects infants of all gestations Necrotizing enterocolitis (NEC) is the least common gastrointestinal disorder seen in the intensive care nursery

The correct answer is -"The development of necrotizing enterocolitis (NEC) occurs due to mesenteric ischemia, bacteria, and feeding." NEC is the most common GI disorder seen in the intensive care nursery and affects approximately 5% of all admissions (7% in infants weighing less than 1500 g). Any conditions or situation that leads to ischemia and bacterial overgrowth in the presence of formula feedings can logically be considered a risk factor. However, prematurity is probably the greatest risk factor of all.

Wenckebach conduction is characterized by: Progressive PR interval prolongation resulting in a blocked or dropped atrial beat Progressive PR interval shortening resulting in a blocked atrial beat Wide QRS complexes

The correct answer is -"progressive PR interval prolongation resulting in a blocked or dropped atrial beat." Wenckebach conduction is characterized by progressive prolongation of the PR interval, which eventually results in a single blocked atrial beat

A C-Reactive Protein (CRP) has a peak response to sepsis at: 24 hours 6 hours 48 hours

The correct answer is 24 hours. CRP is an acute phase reactant that begins to elevate 6-8 hours after onset, and it peaks at 24 hours

Match the condition, patent ductus arteriosus, with one of the characteristics listed below. Systemic hypoperfusion and congestive heart failure with mild or no cyanosis Mild cyanosis caused by complete mixing with normal or increased pulmonary blood flow Acyanotic with no or mild respiratory distress

The correct answer is- "Acyanotic with no or mild respiratory distress." In the neonatal period, most babies with uncomplicated left-to-right shunts including ventricular septal defect and patent ductus arteriosus, are in the group of acyanosis with no or mild respiratory distress.

Pulsus paradoxus may be seen in which of the following conditions? Cardiac tamponade Congestive heart failure Hypovolemic shock

The correct answer is- "Cardiac tamponade." The diagnosis of pericardial tamponade should be suspected in any baby with early signs of poor cardiac output, including tachycardia and poor perfusion. Pulses paradoxus, an exaggerated fall in blood pressure with inspiration, is easiest to observe in neonates with arterial lines for continuous blood pressure monitoring.

An infant presents with signs of congestive heart failure, including a hyperdynamic precordium and hepatomegaly, a cranial bruit, and a systolic ejection murmur. Which of the following defects should be considered? Aortic arch interruption Aortic thrombosis Cerebral arteriovenous malformation

The correct answer is- "Cerebral arteriovenous malformation." Cerebral arteriovenous malformations are manifested in the neonatal period with signs of congestive heart failure, including a hyperdynamic precordium and hepatomegaly. Most defects identified in the neonatal period are vein-of-Galen malformations, although neonates with large pial malformations and congestive heart failure have been described. A cranial bruit and a systolic ejection murmur of increased aortic and pulmonary artery flow are present.

Which of the following cardiac defects should be considered when decreased leg pulses are found on physical examination? Coarctation of the aorta Hypoplastic left heart syndrome Patent ductus arteriosus

The correct answer is- "Coarctation of the aorta." Symptom free newborn infants who have diminished leg pulses on a routine discharge examination have mild coarctation of the aorta and usually no associated aortic valve stenosis or septal defects.

The best description for truncus arteriosus is: Common origin of the aorta and pulmonary artery from a single semilunar valve Connection comprises a group of defects where the pulmonary veins enter the right side of the heart Occurs as an isolated defect or in association with atrial septal defect

The correct answer is- "Common origin of the aorta and pulmonary artery from a single semilunar valve." In truncus arteriosus there is a common origin of aorta and pulmonary artery from a single semilunar valve

The best determination of accurate heart rate on an electrocardiogram (EKG) is: Count all QRS complexes for a period of 6 seconds, and then multiply by 10 Count all QRS complexes for a period of 10 seconds, then multiply by 6 Heart rate cannot be determined accurately by counting only the QRS complexes

The correct answer is- "Count all QRS complexes for a period of 6 seconds, and then multiply by 10." The most accurate method is to determine for determine the heart rate from electrocardiogram. All QRS complexes are counted for a period of 6 seconds, and the rate per minute is obtained by multiplying by 10.

The best way to define shock is: Decreased perfusion to the tissues Low blood pressure Decreased cardiac output

The correct answer is- "Decreased perfusion to the tissues." Shock is a complex state of inadequate circulating blood volume, resulting in insufficient perfusion, oxygenation, and nutrients to tissues.

Appropriate vagal maneuvers for conversion of supraventricular tachycardia (SVT) include which of the following? Elicit diving reflex by placing ice on the infant's head for 15 seconds Carotid massage and orbital pressure Elicit gag with nasogastric (NG) tube

The correct answer is- "Elicit gag with nasogastric (NG) tube." Appropriate vagal maneuvers for conversion of SVT include the following: Elicit a gag with a nasogastric tube, elicit diving reflex by placing a bag filled with ice over the face and ears for 15 seconds. Do not perform carotid massage or apply orbital pressure. Perform vagal maneuvers with an IV line in place; do not continue vagal maneuvers for more than 5 minutes in seriously ill infants before trying other modalities

Which of the following cardiac defects is common in infants with Down Syndrome? Endocardial cushion defects Patent ductus arteriosus Ventricular septal defect

The correct answer is- "Endocardial cushion defects." Endocardial cushion defects are common in babies with Down syndrome, and a large number of these defects are diagnosed as part of the evaluation for Down syndrome in the nursery

Which of the following is true of the hyperoxia test? Generally, shows an increase of >10% SaO2 when pulmonary disease is the primary pathophysiology Is highly accurate to rule out congenital heart disease May show an increase in PaO2 with administration of 100% O2 in the presence of tetralogy of Fallot

The correct answer is- "Generally shows an increase of >10% SaO2 when pulmonary disease is the primary pathophysiology." Patients with pulmonary disease alone generally have a rise in PO2 of greater than 20-30mm Hg or a rise in saturation of greater than 10%. Those with a fixed right-to-left shunt might have a small rise in oxygenation but generally less than these amounts. A fixed right-to-left shunt is a lesion in which no increase is possible either in pulmonary blood flow or in mixing of systemic and pulmonary venous return. Therefore, in such patients, pulmonary venous blood is already nearly completely oxygenated and little additional oxygenation is possible. A good example is in tetrology of Fallot, in which there is both a high resistance to pulmonary flow because of pulmonary stenosis and a right-to-left shunt through a ventricular septal defect. Despite common misconception, the hyperoxia test is not used to rule out congenital heart disease.

Pharmacological management of neonatal hypertension is generally indicated if the infant: Has sustained mean arterial blood pressures, at rest, greater than the 50% Has sustained mean arterial blood pressures, at rest, greater than the 95% Has sustained mean arterial blood pressures, at rest, greater than the 75%

The correct answer is- "Has sustained mean arterial blood pressures, at rest, greater than 95%". Pharmacological therapy is indicated if the infant is symptomatic or if the hypertension is sustained above a mean of 95%, according to gestation and age. Hypertensive crisis is considered when means are > 99% and infant is symptomatic

Infants of mothers with anti-Ro or anti-La antibodies because of lupus erythematosus have a risk of: Persistent pulmonary hypertension of the newborn (PPHN) Ventricular septal defect Heart block

The correct answer is- "Heart block." Mothers with anti-Ro or anti-La antibodies because of lupus erythromatosis have a small (approximately 5%) but important risk of fetal heart block.

Which of the following is true of the association of maternal diabetes mellitus and fetal congenital heart disease? Heart defects are less common in mothers with normal hemoglobin A1c levels Mothers with diabetes present during cardiogenesis have a six-fold risk of having babies with congenital heart disease Poor control, occurring late in gestation, has no effect on the fetal heart, as morphogenesis is essentially complete

The correct answer is- "Heart defects are less common in mothers with normal hemoglobin A1c levels." Mothers who have diabetes mellitus during cardiogenesis have a 3-5 fold risk of having babies with congenital heart disease, most commonly, ventricular septal defect, transposition of the great arteries, tetrology of Fallot, and double-outlet right ventricle. The mechanism of this risk is unknown but presumed to be metabolic abnormalities that occur during critical phases of development. Heart defects are less common in mothers with normal hemoglobin A1C levels. In late gestation, poor control of diabetes increases fetal insulin concentration in response to the increased transplacental glucose load. Insulin is a growth factor for fetal myocardium, and the babies could be born with hypertrophic cardiomyopathy.

The most effective means for increasing cardiac output in the neonate is by increasing: Contractility Preload Heart rate

The correct answer is- "Heart rate." Cardiac output = stroke volume x heart rate. The heart attempts to compensate for decreased cardiac output by increasing heart rate or stroke volume. The fetal myocardium has few contractile elements and poor sympathetic nervous system innervation, making increase in stroke volume difficult. Therefore, the infant increases cardiac output by increasing heart rate.

One common reason for the development of sinus tachycardia includes: Hypertension Maternal lupus High Fever

The correct answer is- "High Fever." Causes of sinus tachycardia include: high fever, pain, hypoxia, exogenous chronotropic agents such as isoproterenol or dobutamine, xanthine therapy, undersedation in paralyzed infants supported by mechanical ventilation, or rarely hyperthyroidism.

The Norwood operation is a palliative procedure used for: Hypoplastic left heart syndrome Aortic stenosis Tricuspid atresia

The correct answer is- "Hypoplastic left heart syndrome." The Norwood palliation is a favored approach in the treatment of hypoplastic left heart. The goal of the Norwood stepwise approach, to the surgical management of hypoplastic left heart syndrome, is first to ensure systemic blood flow and protect the lungs from excessive pulmonary flow and pulmonary hypertension. Subsequently, the systemic venous drainage is separated, providing normal arterial oxygen saturation and ventricular stroke volume.

Which of the following is considered an "acyanotic" congenital heart defect: Transposition of the Great Arteries Truncus arteriosus Patent ductus arteriosus

The correct answer is- "Patent ductus arteriosus." Patent ductus arteriosus is considered an acyanotic congenital heart defect. Generally, cyanosis is not present in an isolated PDA, because the predominant shunt is from left to right

Respiratory distress caused by vascular rings usually indicates severe obstruction. Which of the following causes distress due to obstruction? Pericardial effusion Pulmonary artery sling Pericardial tamponade

The correct answer is- "Pulmonary artery sling." In the first month of life, respiratory distress caused by vascular rings usually indicates severe obstruction. Severe obstruction is caused by either a double aortic arch or a pulmonary artery sling.

Which of the following murmurs is considered non-pathologic in origin? Diastolic murmur Murmur with a thrill Pulmonary flow murmur

The correct answer is- "Pulmonary flow murmur." Innocent murmurs occur in the absence of abnormal cardiac structures. The pulmonary flow murmur is commonly found in low-birth-weight infants. Infants with a pulmonary flow murmur have relative hypoplastic right and left pulmonary arteries at birth, which are a result of the small amount of blood flow during fetal life. The increased flow after birth creates turbulence in the small vessels, which is transmitted along the smaller branches of the pulmonary arteries. The murmur is best heard at the upper left sternal border. The pulmonary flow murmur has a grade of 1/6 to 2/6 intensity, but is transmitted to the right and left chest, both axillae, and back. No other significant cardiac findings are seen. It usually disappears by 3 to 6 months after birth. Persistence beyond this period should lead to further evaluation for anatomic pulmonary artery stenosis

Final diagnosis of Hirschsprung disease is made by: Punch rectal biopsy Radiograph examination Barium enema demonstrating the "funnel" sign

The correct answer is- "Punch rectal biopsy." Definitive diagnosis is made only by suction or punch rectal biopsy through the anus and histologic examination of the specimen obtained. The absence of ganglionic cells in the submucosal plexus firmly establishes the diagnosis of Hirschsprung disease

The differential diagnosis of neonatal hypertension should include which of the following: Renal artery thrombosis Patent ductus arteriosus Hypocalcemia

The correct answer is- "Renal artery thrombosis." The differential diagnosis of neonatal hypertension includes: Aortic arch obstruction (coarctation of the aorta, aortic arch interruption), renal and renovascular problems (renal artery thrombosis or embolus, renal artery stenosis, renal vein thrombosis, cystic renal disease, obstructive uropathy), pharmacologic adverse effects (catecholamines, cocaine, dexamethasone, theophylline) and other causes (exogenous fluid administration, environmental cold or noise stress, seizures, chronic lung disease).

The sound caused by closure of the aortic and pulmonary valves, which mark the end of systole and beginning of ventricular diastole, is: S1 S2 S3

The correct answer is- "S2." S2 is the sound created by closure of the aortic and pulmonary valves, which marks the end of systole and the beginning of ventricular diastole. S2 is best heard in the upper left sternal border or pulmonic area.

Match the condition, tricuspid atresia, with the one of the characteristics listed below. Severe cyanosis caused by separate circulations and poor mixing Mild cyanosis caused by complete mixing with normal or increased pulmonary blood flow Severe cyanosis caused by restricted pulmonary blood flow

The correct answer is- "Severe cyanosis caused by restricted pulmonary blood flow." Tricuspid atresia usually results in complete absence of the components of the tricuspid valve, with a smooth floor of the right atrium at the usual location of the tricuspid valve. In rare instances, tricuspid valve tissue and chordae are present, but the valve is imperforate. Associated right ventricular hypoplasia and pulmonary artery hypoplasia are proportional to the size of the ventricular septal defect and the degree of subpulmonary and pulmonary valve stenosis.

Which type of cardiac arrhythmia is most often associated with Wolfe-Parkinson-White syndrome (WPW)? Atrial ectopic tachycardia Supraventricular tachycardia (SVT) Complete Heart Block

The correct answer is- "Supraventricular tachycardia (SVT)." Of the infants with SVT, 10% to 50% have Wolfe-Parkinson-White (WPW) syndrome in which the atrial impulse activates the whole or some part of the ventricle or the ventricular impulse activates the whole or some part of the atrium earlier than would be expected if the impulse traveled via the normal pathway.

Which of the following cardiac defects consist of a ventricular septal defect (VSD), pulmonary stenosis, or some other right ventricular outflow tract obstruction, an overriding aorta, and a hypertrophied right ventricle? Endocardial cushion defect Truncus arteriosus Tetralogy of Fallot

The correct answer is- "Tetralogy of Fallot." TOF consists of a large VSD, pulmonary stenosis or other right ventricular outflow tract obstruction, overriding aorta and hypertrophied right ventricle. The right ventricle may not be hypertrophied initially. Pulmonary valve atresia is seen in the most severe form of TOF

The congenital heart defect where the aorta connects to the right ventricle and the pulmonary artery connects to the left ventricle is known as: Transposition of the great arteries Truncus arteriosus Total anomalous pulmonary venous return

The correct answer is- "Transposition of the great arteries." In Transposition of the great arteries, the aorta arises from the desaturated right ventricle and the pulmonary artery from the systemic, oxygenated left ventricle, causing deoxygenated blood to recirculate to the body and highly oxygenated blood to return to the lungs

Common treatment for sinus tachycardia includes which of the following: Orbital pressure Vagal maneuvers Treatment of the underlying cause

The correct answer is- "Treatment of the underlying cause." Vagal maneuvers and other methods of converting supraventricular tachycardia will not terminate sinus tachycardia. Vagal maneuvers can briefly slow the rhythm, but because the underlying cause of sinus tachycardia is unaffected by such maneuvers, the rhythm resumes immediately.

The congenital heart defect, Tetralogy of Fallot, includes which of the following defects: A large aorta overriding the atrial septum Valvular pulmonary stenosis Left ventricular hypertrophy

The correct answer is- "Valvular pulmonary stenosis." Tetralogy of Fallot includes subvalvular and valvar pulmonary stenosis, a large ventricular septal defect, a large aorta overriding the ventricular septum, and right ventricular hypertrophy. A variable degree of pulmonary artery hypoplasia, an atrial septal defect or foramen ovale, patent ductus arteriosus, or systemic-to-pulmonary collateral vessels are often present. In the most severe form of tetralogy of Fallot, atresia of the pulmonary valve or of the entire right ventricular outflow tract (OFR) and main pulmonary artery occurs

The criteria for the diagnosis of sinus tachycardia include which of the following: Abnormal P wave axis Rapid onset and termination Rates less than 220 beats per minute

The correct answer is- "rates less than 220 beats per minute." The criteria for sinus tachycardia are normal P wave axis, gradual onset and termination, and rates less than 220 beats per minute.

When determining heart size using an anteroposterior chest film, the width of the cardiothymic silhouette compared with the width of the chest at its largest dimension should have a ratio of: 0.30 or less 0.60 or less 0.75 or more

The correct answer is-"0.60 or less." For the determination of heart size, the width of the cardiothymic silhouette is measured and compared with the width of the chest at its largest dimension. The ratio should be 0.60 or less.

Which radiographic finding is associated with obstructed total anomalous pulmonary venous return? Massive cardiomegaly Right aortic arch Pulmonary edema

The correct answer is-"Pulmonary edema." Distinctive radiographic finding of Obstructed total anomalous pulmonary venous drainage is pulmonary edema related to pulmonary venous congestion.

Which of the following cardiac defects is the most common cyanotic heart defect identified in the first week of life? Transposition of the great arteries Tetralogy of Fallot Tricuspid atresia

The correct answer is-"Transposition of the great arteries." Transposition of the great arteries is the most common cyanotic heart defect identified in the first week of life, and the diagnosis should be considered in any cyanotic neonate

HSV-2 caused encephalitis generally presents at: 16-19 days of life 10-12 days of life 5-7 days of life

The correct answer is: 16-19 days of life. HSV-2 encephalitis presents around 16-19 days of age. Disseminated infection generally presents earlier, at 10-12 days of age, and skin, eye, mucous membrane infection presents at 10-11 days of age.

The standard dose of oral elemental iron (in ferrous sulfate) recommended for healthy premature newborns is: 1 mg/kg/day 2-4 mg/kg/day 5-6 mg/kg/day

The correct answer is: 2-4 mg/day. The dose of oral elemental iron (in ferrous sulfate) recommended by the American Academy of Pediatrics is 2-4 mg/kg/day of elemental iron, to be started by 1 month of age. If there is iron deficiency anemia, the dose may be increased to 6 mg/kg/day.

A 1.8 kg neonatal patient with an intravascular thrombus is to receive a standard dose of Enoxaparin (low molecular weight heparin). The correct dose to order would be: 1.5 mg subcutaneous Q 12 hrs. 2.7 mg subcutaneous Q 12 hrs. 3.6 mg intramuscular Q 12 hrs.

The correct answer is: 2.7 mg subcutaneous Q 12 hrs. The standard enoxaparin (low molecular weight heparin) dose is 1.5 mg/kg subcutaneous Q 12 hrs. Some patients may have a higher initial dose given

A capillary hematocrit may be up to how much higher than a venous specimen? 20% 30% 40%

The correct answer is: 20%. A capillary hematocrit may be 5-20% higher than a venous specimen, and treatment for a high hematocrit should never be based on a capillary specimen

The risk of development of chronic epilepsy in a neonate with seizures is: 30% or less 50%-75% 100%

The correct answer is: 30% or less. Neonates with seizures do have a risk of epilepsy during childhood and later adult life. However, the risk is generally 30% or less, particularly if they are inborn

The onset of vesicular lesions in a newborn whose mother had varicella zoster at 37 weeks gestation can be expected at: 1-3 days of age 5-10 days of age 14-21 days of age

The correct answer is: 5-10 days of age. Primary varicella may occur in the neonate when the mother has had a primary varicella infection in the last 3 weeks of pregnancy. The onset of diffuse vesicular lesions is 5 to 10 days of age.

The protective acid mantle of the epidermis takes how long to develop after birth? 3-4 weeks 5-6 weeks 7-8 weeks

The correct answer is: 5-6 weeks. The acid mantle of the epidermis develops over 5-6 weeks. Newborn skin has a pH around 6.0, dropping to 5.1 over 5-6 weeks.

The red blood cell lifespan in a healthy term newborn is: 30-50 days 60-80 days 90-120 days

The correct answer is: 60-80 days. The normal red blood cell lifespan in a healthy term newborn is 60-80 days. In contrast, the lifespan for adult red blood cells is 120 days, and for ELBW infants it is 30-50 days.

A platelet transfusion of 15 ml/kg will increase a patient's platelet count by: 70,000-100,000 120,000-150,000 120,000-150,000

The correct answer is: 70,000-100,000. They have a half-life of 1-2 days, and will essentially be all gone by 5-6 days post-transfusion.

The hemoglobin nadir in a healthy term infant occurs at: 4-6 weeks of age 8-12 weeks of age 14-18 weeks of age

The correct answer is: 8-12 weeks of age. The nadir hemoglobin in healthy term infants occurs at 8-12 weeks of age secondary to changes in tissue oxygenation and erythropoietin production.

The water content of vernix caseosa is approximately: 30% 50% 80%

The correct answer is: 80%. Vernix caseosa is a mixture of lipids of sebaceous origin (wax esters, cholesterol, ceramides and other lipid components), corneocytes and is about 80% water. It possesses both emollient and cleansing functions.

Holoprosencephaly is associated with which of the following findings? A single sphere cerebral structure with a common ventricle Absence of midline facial anomalies Macrocephaly

The correct answer is: A single sphere cerebral structure with a common ventricle. Alobar holoprosencephaly is the most common type and the most severe. The complete absence of cleavage of the prosencephalon results in a single cerebral sphere surrounding a lone central ventricle.

A key step in colonization of the premature neonate with Candida is: Rapid reproduction of the organism Toxin release Adhesion to cells

The correct answer is: Adhesion to cells. Candida species can adhere to epithelial and endothelial cells, as well as devices such as catheters and form a biofilm which protects the organism from host response.

A newborn is noted to have several hemangiomas in a beard-like pattern over the lower face. This infant is at risk for: Airway hemangiomas Cardiac vascular malformation Cervical spinal involvement

The correct answer is: Airway hemangiomas. While most cutaneous hemangiomas are benign, some can indicate further problems. Sixty percent of infants with multiple facial hemangiomas distributed in a beard-like pattern develop symptomatic airway hemangiomas.

Which lab test would you order to monitor the patient receiving Enoxaparin (low molecular weight heparin? Fibrinogen level Anti-factor Xa level Platelets

The correct answer is: Anti-factor Xa level. Anti-factor Xa level should be monitored 4-6 hours after a dose of Enoxaparin is given.

Harlequin color change in the newborn is reflective of imbalance in: Autonomic regulation Peripheral innervation Cutaneous vessel immaturity

The correct answer is: Autonomic regulation. Harlequin color change is demonstrated when the dependent portion of the body becomes red, the upper portion becomes pale, and a sharp demarcation is evident between the two areas. It occurs due to a temporary imbalance in autonomic regulation of cutaneous vessels.

The inheritance pattern for cutis laxa is: Autosomal recessive only Autosomal dominant only Both autosomal dominant and autosomal recessive

The correct answer is: Both autosomal dominant and autosomal recessive. Cutis laxa (congenital elastosis) has three major forms, with one inherited as an autosomal dominant trait, and two inherited as an autosomal recessive trait.

A sebaceous nevus: Can enlarge at puberty Contains hair when on the scalp Most often is found on the extremities

The correct answer is: Can enlarge at puberty. The sebaceous nevus is most often found on the face or scalp. It is devoid of hair when occurring on the scalp because the hair follicles are rudimentary. It may remain unchanged until puberty, when it enlarges and becomes thicker due to stimulation of the sebaceous glands.

Alpha 1-Antitrysin deficiency in the neonate causes: Hypoglycemia Emphysema Cirrhosis

The correct answer is: Cirrhosis. Alpha 1-Antitrypsin deficiency is the most common inherited cause of neonatal hepatitis syndrome. It can cause cirrhosis in the neonate and can cause emphysema in the adult (not the neonate). Outcome is based on the severity of neonatal liver disease. 5-10% of children will require liver transplant, and 50% of children are clinically normal by age 10.

Which of the following statements about seizures in infants is true? Clonic seizures have a rapid phase followed by slow return movement Myoclonic seizures are always generalized Seizure onset after 4 days of life generally has a poor prognosis

The correct answer is: Clonic seizures have a rapid phase followed by slow return movement. Myoclonic seizures may be generalized, multifocal or focal. Seizure prognosis depends on several factors, including underlying etiology.

Infants with dystrophic epidermolysis bullosa are at risk for: Cardiac dysrhythmias Hearing loss Clotting abnormalities

The correct answer is: Clotting abnormalities. All forms of epidermolysis bullosa may result in permanent scarring with resultant loss of function. In addition, complications may include dysphagia, anemia, delayed physical and sexual development, visceral amyloidosis, hyperglobulinemic purpura, clotting abnormalities and squamous cell carcinoma.

A neonate presents with an extensive diffuse rash at 12 hours of age consisting of macules, papules and pustules, including the palms and soles of the feet. This is consistent with: Congenital cutaneous candidiasis Erythema toxicum neonatorum Primary cutaneous aspergillosis

The correct answer is: Congenital cutaneous candidiasis. Congenital cutaneous candidiasis presents by 12 hours of age with an extensive diffuse rash which includes macules, papules and pustules, and this rash frequently involves the palms and the soles of the feet. Primary cutaneous aspergillosis presents a little later, and generally presents with plaque formation with eschar

Persistent cutis marmorata is common in which disorder? Beckwith-Wiedemann syndrome CHARGE association Cornelia de Lange syndrome

The correct answer is: Cornelia de Lange syndrome. While cutis marmorata, the temporary mottling of the skin with constriction of blood vessels, commonly seen in the newborn with chilling or stress, persistent cutis marmorata is seen in infants with Cornelia de Lange syndrome, as well as in infants with Trisomy 18 or Trisomy 21

Over the first week of life the reticulocyte count in a healthy newborn will: Decrease Increase Remain unchanged

The correct answer is: Decrease. The reticulocyte count at birth is inversely proportional to gestational age at birth, and it falls to less than 2% by 7 days of age.

Most infants with harlequin ichthyosis: Die in the neonatal period Die in early childhood Survive into adulthood

The correct answer is: Die in the neonatal period. Most newborns with harlequin ichthyosis are born prematurely, between 32 and 35 weeks gestation. This disorder is associated with severe complications such as sepsis and gangrene. Most infants die in the neonatal period.

The most common cause of excessive scaling and desquamation in the newborn is: Dysmaturity Congenital ichthyosis Ectodermal dysplasia

The correct answer is: Dysmaturity. Dysmaturity of the skin in infants born at greater than 40 weeks gestation is the most common cause of excessive skin scaling and desquamation.

The nerve networks in the skin develop at which time in gestation? Early embryonic period By the end of the first trimester Toward the end of the second trimester

The correct answer is: Early embryonic period. Nerve networks in the dermis develop at a very early embryonic age, and are distributed randomly.

While performing a newborn exam, you find that the neonate has hyperextensible skin that snaps back into place, and joint laxity. The most likely etiology is that the neonate has: Ehler-Danlos Syndrome Neonatal porphyria Cutis laxa

The correct answer is: Ehler-Danlos Syndrome. Ehler-Danlos Syndrome refers to a group of inherited connective tissue disorders which feature hyperextensible skin, joint laxity and soft tissue fragility. In contrast to cutis laxa, the skin snaps back into place

A preterm infant with necrotizing enterocolitis has bleeding from an endotracheal tube, and the abdominal wound, in addition to significant bruising of the skin. You suspect DIC. Which lab findings would support this diagnosis? Elevated fibrin degradation products Decreased D-Dimer level Increased fibrinogen level

The correct answer is: Elevated fibrin degradation products. Laboratory abnormalities seen with DIC are low platelets, elevated fibrin degradation products or D-Dimers, prolonged PT, prolonged aPTT, prolonged clotting time and low fibrinogen. Also Factors VIII and II are decreased, as well as protein C and protein S.

A newborn is noted to have symmetric salmon-colored cutaneous lesions on the eyelids and glabella that blanch with pressure. These most often: Fade by one year Are permanent Resolve by one month

The correct answer is: Fade by one year. The nevus simplex is the most common neonatal cutaneous lesion. It is a salmon-colored patch, usually centrally located on the nape of the neck, the eyelids and/or glabella. They usually fade by one year of age, although those on the nape of the neck may be more persistent.

In Disseminated Intravascular Coagulation (DIC) microthrombi are formed by: Protein degradation Platelets Fibrin strands

The correct answer is: Fibrin strands. In DIC, fibrinogen converts to fibrin, and fibrin strands cause red blood cell fragmentation and microthrombi.

Cryoprecipitate is given to patients with DIC as it contains: Factors V and VII Antithrombin III Fibrinogen and Factor VIII

The correct answer is: Fibrinogen and Factor VIII. Cryoprecipitate is a concentrated source of Factor VIII, fibrinogen and Factor XIII,

Abnormal hyperpigmentation of the areolas and genitals in a newborn may indicate in utero: Glucocorticoid insufficiency Estrogen excess Androgen deficiency

The correct answer is: Glucocorticoid insufficiency. Abnormal hyperpigmentation of the areolas and genitals, although rare, may indicate the presence of in utero glucocorticoid insufficiency associated with defects in hydrocortisone biosynthesis.

A 32-week black neonate is admitted to the NICU for care. His mother is critically ill with pneumonia, and received a dose of methylene blue prior to delivery for methemoglobinemia. The neonate's first bilirubin at 6 hours of age is 23 mg/dL, and his hematocrit is 28%. You suspect the cause of this hyperbilirubinemia is: Glucose-6-phosphate dehydrogenase (G6PD) deficiency Sickle cell disease Hereditary elliptocytosis

The correct answer is: Glucose-6-phosphate dehydrogenase (G6PD) deficiency. Neonates with G6PD deficiency can present with early onset hyperbilirubinemia and anemia. Exposure to certain drugs, including methylene blue (as received by this mother), can precipitate hemolysis

A preterm infant born to a Hepatitis B positive mother should receive: HBIG immediately and Hepatitis B vaccine at term, 1 month and 6 months of age HBIG within 12 hours and Hepatitis B vaccine at birth, with a subsequent 3 dose course HBIG within 12 hours and Hepatitis B vaccine at 1, 2 and 6 months of age

The correct answer is: HBIG within 12 hours and Hepatitis B vaccine at birth, with a subsequent 3 dose course. A preterm infant needs Hepatitis B immunization at birth, and HBIG within 12 hours of birth. The initial course should not be counted as part of the 3 dose course given to infants

A neonate is noted to have a very large congenital melanocytic nevus approximately 22 cm in diameter extending from the chest to the lower back. You should explain to the mother that it: Can grow over the first year of life Fades over time Has a risk of malignant transformation

The correct answer is: Has a risk of malignant transformation. Congenital melanocytic nevi represent a nested proliferation of melanocytes that may be present at birth. Large congenital melanocytic nevi (>20 cm) have a 6-8% of malignant transformation. These lesions are often surgically excised or treated with laser therapy, and should be monitored for malignancy over time

Linear skull fractures in the newborn most often: Heal spontaneously Require emergent surgery Require surgery in the first 24-48 hours of life

The correct answer is: Heal spontaneously Linear skull fractures generally heal spontaneously with no sequelae unless underlying cerebral damage or a depressed fracture is present.

The primary hemoglobin in the neonate is: Hemoglobin A Hemoglobin S Hemoglobin F

The correct answer is: Hemoglobin F. Hemoglobin F (fetal hemoglobin) is the predominate hemoglobin in the fetus and neonate. The shift to adult hemoglobin (hemoglobin A) occurs by 6 months of age.

An infant is born with low birth weight, microcephaly, chorioretinitis and numerous congenital scars. This may represent first trimester infection with: Herpes simplex virus Toxoplasmosis gondii Treponema pallidum

The correct answer is: Herpes simplex virus. Maternal herpes simplex viral infection in the first trimester may occur early in gestation, resulting in a neonate with low birth weight, microcephaly, chorioretinitis and congenital scarring.

A term newborn born to a mother after an uneventful pregnancy, labor and delivery is transferred to the NICU for a seizure and unexplained petechiae. The admission CBC shows a platelet count of 17,000. To confirm your suspicion of neonatal alloimmune thrombocytopenia you would test for: Human platelet antigen Low fibrinogen level Factor VIII deficiency

The correct answer is: Human platelet antigen. The most common cause of severe neonatal thrombocytopenia and intracranial hemorrhage in term neonates is fetal and neonatal alloimmune thrombocytopenia. Because it is caused by the transplacental passage of maternal antibodies of certain human platelet antigens (HPAs), testing for human platelet antigens in the infant and mother would support the diagnosis.

Erythropoietin production increases in response to: Hypertension Acidosis Hypoxia

The correct answer is: Hypoxia. Erythropoietin production is sensitive to hypoxia, and in response to hypoxic conditions erythropoietin production increases

Incontinentia pigmenti is marked by clear to yellow pustules presenting at birth that appear: Confined to mucous membranes In a linear pattern Scattered in a pinpoint pattern

The correct answer is: In a linear pattern. The cutaneous lesions of Incontinentia pigmenti present at birth as clear to yellow vesicles arranged in a linear pattern on the trunk and limbs, and appear in crops over the first few weeks of life

Junctional epidermolysis bullosa (Herlitz): Is a very severe form of the disease Usually presents at 1-2 months of age Has an autosomal dominant inheritance pattern

The correct answer is: Is a very severe form of the disease. Junctional epidermolysis bullosa (Herlitz) is most often present at birth, and is a very severe form of the disease with bullae formation and loosening of the epidermis after minimal trauma. It has an autosomal recessive inheritance pattern

In comparison to bovine based formula, the Vitamin K content of human breast milk is: More than bovine formula Less than bovine formula The same as bovine formula

The correct answer is: Less than bovine formula. Human breast milk is relatively low in Vitamin K (10mcg/L), in comparison to bovine-based formula 65-100 mcg/L).

Between weeks 6 and 16 of gestation the major site of fetal hematopoiesis is the: Liver Bone marrow Thymus

The correct answer is: Liver. The site of hematopoiesis is unknown in the earliest embryonic period, but between weeks 6 and 16 the fetal liver is the major site of hematopoiesis.

The enzyme present in breast milk that serves in host defense is: Amylase Trypsin Lysozyme

The correct answer is: Lysozyme. Lysozyme is an active component of human breast milk. It lyses the cell walls of many bacteria.

The most effective cranial imaging for a neonate with an asphyxial insult is cranial: Ultrasound Magnetic resonance imaging (MRI) Computed tomography (CT)

The correct answer is: Magnetic resonance imaging (MRI). MRI is more sensitive that CT in detecting changes in the central gray nuclei, the most frequent problem with hypoxic-ischemic encephalopathy, and it can show different patterns of injury. Cranial ultrasounds are not good at finding subtle changes, although they may identify cerebral edema by measurement of ventricular size

A newborn presents with a facial port-wine stain over the first branch of the trigeminal nerve. Which imaging study is recommended to further evaluate this infant? Magnetic resonance imaging X-ray of the skull Arteriogram

The correct answer is: Magnetic resonance imaging. A facial port-wine stain distributed over the first branch of the trigeminal nerve is indicative of Sturge-Weber syndrome, which is a leptomeningeal malformation. It is associated with seizures, developmental delay, contralateral hemiparesis and ipsilateral intracortical calcifications. The diagnostic imaging study of choice is magnetic resonance imaging of the head

A newborn with alloimmune thrombocytopenia is to be transfused with platelets, but no HPA-specific platelets are available. The newborn should be transfused with: Maternal washed platelets O negative irradiated platelets Random donor platelets

The correct answer is: Maternal washed platelets. If HPA specific platelets are not available, the newborn should be transfused with washed maternal platelets is AB-negative plasma. Transfusion of random donor platelets rarely results in sustained improvement in the platelet count due to the continued antibody destruction.

Neonatal urinary tract infection (UTI): May be acquired hematogenously Can be diagnosed by a bagged specimen if adequate prepping has occurred Is more common in females than males

The correct answer is: May be acquired hematogenously. Neonatal UTI (in the first month of life) may be acquired through vertical transmission, or hematogenously from bacteremia. It is more common in male neonates than females. It cannot be reliably diagnosed by culture from a bagged specimen; the specimen must be obtained by suprapubic aspiration or catheterization

A newborn is noted to have 14 large café-au-lait spots scattered over the trunk and axillae with freckling in the flexure creases, but no white macules. This finding raises the concern for: Neurofibromatosis Incontinentia pigmenti Tuberous sclerosus

The correct answer is: Neurofibromatosis. Café-au-lait spots are a normal variation in pigmentation. However, if there are more than 6 in number, and if they are large and accompanied by freckling (small café-au-lait spots) in the flexure creases, this may represent neurofibromatosis. Patients with tuberous sclerosis may also have these café au lait spots, but they are accompanied by white macules.

An African-American neonate presents at birth with superficial vesiculopustular lesions, which, when ruptured, leave a small pigmented macule surrounded by a ring of very fine white scales. These require: No treatment Antibiotic ointment Culture and systemic antifungal therapy

The correct answer is: No treatment. These lesions are a result of pustular melanosis. They are common in the neonate, and more frequent in African- American neonates. The lesions rupture at 12-48 hours, leaving small, pigmented macules with a fine, white scales. The condition is benign, requiring no treatment.

Miliaria crystallina develops secondary to: Obstructed sweat glands Sebaceous gland hyperplasia Excoriation of the skin

The correct answer is: Obstructed sweat glands. Miliaria crystallina occurs because of eccrine sweat duct obstruction, creating small, thin-walled vesicles without inflammation. No therapy is needed

The most common cause of neonatal seizures is: Bacterial infection Metabolic disturbance Perinatal hypoxia-ischemia

The correct answer is: Perinatal hypoxia-ischemia. Perinatal hypoxia-ischemia is the most common causal factor, with most asphyxia occurring before or during the birth process.

A Wright's stain in an infant with pustular melanosis will reveal: Polymorphonuclear leukocytes Staphylococcus species Eosinophils

The correct answer is: Polymorphonuclear leukocytes. A Wright's stain of pustular melanosis lesions will reveal polymorphonuclear leukocytes and no organisms. Eosinophils will be found with the lesions of erythema toxicum neonatorum.

Periventricular leukomalacia causing spastic diplegia in the premature neonate is secondary to damage to the: Cerebellum Parasagittal area Pyramidal tracts

The correct answer is: Pyramidal tracts. Periventricular leukomalacia involving the pyramidal tracts usually results in spastic diplegia or quadriplegic forms of cerebral palsy.

Neurological assessment is best performed when the infant is in which state? Active alert Quiet alert Sleeping

The correct answer is: Quiet alert. The neonate's state should be noted both before and during the examination, which optimally is performed with the neonate in the quiet alert state. Timing the examination for 30 minutes to one hour before a feeding may increase the chance of the neonate being in this state.

A 10 day old critically ill premature newborn gradually develops diffuse hardening of the subcutaneous tissue and tight smooth skin which is non-pitting, cold and waxy hard. This likely represents: Sclerema neonatorum Subcutaneous fat necrosis Scleredema

The correct answer is: Sclerema neonatorum. The etiology of sclerema neonatorum is unclear, but it occurs most frequently in the very ill premature infant, and presents with diffuse hardening of the subcutaneous tissue, resulting in the described findings. It has a high mortality rate.

A NICU patient is noted to have significant erythema on the face, which gradually spreads downward, with a fever and gradual development of bullous lesions in flexural areas and resultant shedding of the entire epidermis of the area. In addition to isolation, you order antibiotic therapy with a: Carbapenem Cephalosporin Semi-synthetic penicillin

The correct answer is: Semi-synthetic penicillin. The described presentation is consistent with Staphylococcus Scalded Skin Syndrome caused by staphylococcus aureus. The recommended treatment is a semi-synthetic penicillin

A large indurated red plaque with a sharply circumscribed border and a subcutaneous nodular feel on palpation is noted on the buttocks of a large for gestational age infant at one week of age. Which lab test should be checked to monitor for a complication of this condition? Blood culture Glucose level Serum calcium level

The correct answer is: Serum calcium level. The lesion described is a subcutaneous fat necrosis, which presents as a hard, circumscribed, red, or purple nodule under the dermis in the subcutaneous tissue. The affected fat is firm, and may have an orange-peel texture. Nodules can appear on the trunk, extremities, or face, usually during the first 1-4 weeks of life. They are a result of cold exposure, trauma, asphyxia or peripheral circulatory collapse. They may grow larger initially and then resolve spontaneously within several weeks or months. It is associated with hypercalcemia, and thus serum calcium levels should be monitored

The hemoglobin nadir for preterm infants occurs at an earlier age because of: Immature bone marrow activity Inadequate renal response to hypoxia Shorter red blood cell lifespan

The correct answer is: Shorter red blood cell lifespan. Shorter red blood cell lifespan in preterm newborns with lower EPO levels results in a hemoglobin nadir that occurs earlier, at 6 weeks of age, and is lower than in a healthy term newborn

Rifampin may be added to therapy when other medications have been unable to clear which type of infection? Staphylococcal sepsis Gram negative sepsis Fungal sepsis

The correct answer is: Staphylococcal sepsis. Rifampin is used in combination with vancomycin or aminoglycosides for treatment of persistent staphylococcal infections. It is also used as prophylaxis against infections caused by N. meningitidis and H. influenza type b.

If a neonate has a persistent candida diaper rash which has not resolved with a topical antifungal, the next appropriate treatment step would be to: Add a topical steroid to the topical antifungal Draw a fungal blood culture and start systemic therapy Start an oral antifungal medication

The correct answer is: Start an oral antifungal medication. If a candida diaper dermatitis is persistent or severe, adding an oral antifungal medication may be recommended to clear or prevent reinfection from candida in the GI tract.

A newborn presents at birth with annular skin lesions on the face and scalp, and a "racoon eyes" appearance is also noted. The mother of this infant should be tested for: Systemic Lupus Erythematosus Syphilis Porphyria

The correct answer is: Systemic Lupus Erythematosus (SLE). Neonatal lupus erythematosus is an acquired autoimmune disease caused by transplacental passage of maternal SLE autoantibodies. The classic skin lesions in the newborn are annular lesions on the face and scalp, and there may be a "racoon eyes" appearance. Up to 40-60% of mothers of affected infants are asymptomatic. Newborns with SLE may develop rheumatic disease later in childhood.

In an infant who has an acute hemorrhage, the blood will be reflected in the laboratory hematocrit: Three to four hours after blood loss Within 2 hours of blood loss Immediately

The correct answer is: Three to four hours after blood loss. Anemia usually develops three to four hours after blood loss. Repeat testing six to twelve hours later will confirm the true extent of the loss.

Xeroderma pigmentosum results in extreme sensitivity to: Adhesive products Skin cleansers Ultraviolet light

The correct answer is: Ultraviolet light. Xeroderma pigmentosum is a skin disorder transmitted by an autosomal recessive gene that results in extreme sensitivity of the skin to ultraviolet light.

A physical exam finding consistent with cephalohematoma is: Craniotabes Edema that crosses suture lines Well-demarcated edges

The correct answer is: Well-demarcated edges. Cephalohematoma is associated with the use of forceps during delivery. Cephalohematoma is located below the periosteum and confined by the cranial sutures creating well-demarcated edges. The volume of blood contained within is limited because the periosteum of the cranial bone limits the potential space available for blood to expand. Cephalohematoma may contribute to jaundice, but is rarely of neurologic significance.

An HIV-exposed neonate should have which test done to definitively determine HIV status? p24 antigen Enzyme immunoassay DNA polymerase chain reaction (PCR)

The correct response is: DNA polymerase chain reaction (PCR). Diagnosis can be made in the neonate by viral cultures polymerase chain reactions (PCR), or viral load testing. Routine HIV antibody testing by the enzyme immunoassay or Western blot cannot be used because of placental transfer of maternal IgG antibodies. Use of the p24 antigen test for HIV 1 is not recommended because of poor sensitivity.

A recommendation for hand hygiene in the nursery is: Use of alcohol based products as an adjunct to hand hygiene Initial 5 minute surgical scrub A 10 second scrub before and after handling each infant

The current recommendations for handwashing in the NICU include: Removal of all rings; no nail polish or false nails Initial 3-minute scrub to the elbow. A 15 second scrub before and after handling each infant Use of alcohol-based hand gels before and after handling infant if the hands are not visibly soiled.

The recommended minimal length of antibiotic therapy for acute osteomyelitis is: 10 days 14 days 21 days

The durations of treatment should be a minimum of 3 weeks (21 days) to not more than 6 weeks.

Primitive or primary reflexes include the following: Galant and Moro Head righting and Toe Grasp Parachute and Head Lag

The following are developmental reflexes and are sometimes referred to as primary or primitive reflexes: Suck, Rooting, Palmer grasp, Tonic neck, Moro, Stepping, Truncal incurvation (Galant), and Babinski. Primative or primary reflexes do not require functional brain above the diencephalon

The four (4) legal elements necessary to prove in a malpractice case include: Duty, breach of ethics, injury, causation Duty, breach of duty, injury, causation Duty, care, injury, ethical obligation

The following are the four elements of necessary to prove negligence: 1.The nurse had a duty to her or his patient, 2. There was breach of that duty, 3. Harm or damages did occur to the patient, and 4. Breach of that duty resulted in harm (proximal cause).

The recommended dose for Acyclovir is: 10 mg/kg/dose every 8 hours 20 mg/kg/dose every 8 hours 30 mg/kg/dose every 8 hours

The generally recommended dose for Acyclovir is 20 mg/kg/dose every 8 hours for 14-21 days. If there is renal impairment dosing interval is prolonged based on the serum creatinine.

Conjugated bilirubin can be converted back to unconjugated bilirubin in the: Colon Rectum Small intestine

The high concentration of β-glucuronidase in the brush border of the small intestine of the newborn can convert conjugated bilirubin into unconjugated bilirubin. This can be absorbed back into the portal circulation.

Which statement below best describes the post discharge follow up timeline for infants diagnosed with congenital syphilis? No follow up is indicated if fully treated 6 months, 12 months, 18 months after completion of therapy 3 months, 6 month, 12 months after completion of therapy

The infant should have repeated quantitative NTA tests at 3, 6, and 12 months. Most infants have a negative titer with adequate treatment. A rising titer requires further investigation and retreatment

Pharmacogenomics

The marriage of pharmacology with genomics and is considered a broader scope Study of genome-wide response to small molecular weight compounds administered with therapeutic intent Finding the right drug for the right disease

The most common cause of lower respiratory tract infection in children less than one year of age is: Chlamydia pneumonia Respiratory syncytial virus Beta strep pneumonia

The most common cause of lower respiratory tract infection in infants <1 year of age is respiratory syncytial virus, with up to 3% of infants hospitalized for RSV in the first year of life.

The most common organism associated with urinary tract infections in the newborn is: E-coli Coagulase negative staph Klebsiella

The most common organism causing UTI in neonates is E. coli, which accounts for 91% of community-acquired infections in children younger than 8 weeks.

Which of the following cardiac defects is found with VACTERL association? Coarctation of the Aorta Truncus arteriosus Atrial septal defect

The preferred response is "Coarctation of the aorta". The VACTERL association describes a series of defects, including vertebral abnormalities, anal atresia, tracheoesophageal fistula, renal abnormalities, congenital heart disease, and limb abnormalities. Seventy percent (70%) of infants with VACTERL have three affected systems, 25% have four components, and 10% have five. Variability of components is vast, and congenital heart disease is present in about 75% of these infants. The most common heart defects are ventricular septal defect (VSD), Tetralogy of Fallot, and coarctation of the aorta

The primary abnormality in hydrancephaly is absence of the: Cerebellum Cerebral hemispheres Midbrain

The primary abnormality in hydrancephaly is absence of the cerebral hemispheres, and may be due to a congenital infection or ischemic brain necrosis. An infant with hydrancephaly may appear normal at birth, and be diagnosed at a later age when developmental milestones are missed

The primary action of phototherapy is: Photoisomerization of bilirubin to a water-soluble product Oxidation of the bilirubin molecule Rapid reversal of enterohepatic absorption

The primary action of phototherapy is to cause photoisomerization of the bilirubin molecule, thus changing it to a water-soluble compound which can be eliminated. Oxidation also occurs, but plays a very minor role

A positive Babinski reflex in the neonate is observed when there is: Extension or flexion of the toes Inward movement of the foot Plantar flexion

The response to stimulation of the sole (Babinski reflex) in the neonatal period is reflex is positive if extension and flexion of the toes occurs. Babinski onset is at about 34 weeks, and it disappears by 14 months. Absent Babinski may indicate CNS depression or abnormal spinal innervation.

The Candida characteristic that allows it to progress from colonization to dissemination before clinical signs are noted is: Gut translocation Rapid replication Slow growth

The slow growing nature of Candida species allows it to progress from colonization, which is common, to systemic dissemination before clinical signs and symptoms are seen

The IgG level of a term neonate is: Higher than the mother's level Lower than the mother's level The same as the mother's level

The transfer of IgG from mother to fetus is an active placental process, and the term neonate's IgG level is 5-10% higher than the mother's.

The transition from embryonic to fetal epidermal layers occurs at: 30 days 60 days 90 days

The transition of the epidermis from embryonic to fetal stratification occurs around 60 days of age. There is a formation of intermediate layers of cells, and presence of new keratin filaments, with eventual transition of the epidermis into several distinct layers.

Myelination of the brain: Advances at different rates in different regions of the brain Begins at 10 weeks gestation Is complete by 40 weeks gestation

There is no apparent myelination of the forebrain before 7 months of gestation. The rate of myelination varies with the brain structure. Myelination occurs over a long time, in some cases into adulthood.

Eye drainage in the newborn noted from 2-5 days of life is consistent with: Gonorrheal conjunctivitis Chlamydia trachomatis Chemical conjunctivitis

Time of onset for conjunctivitis secondary to gonorrhea in the newborn is 2-5 days of age. Chemical conjunctivitis occurs in the first 48 hours of life. The onset of chlamydial conjunctivitis is generally 5-14 days of age

To meet the legal standard of informed consent, the professional obtaining the consent should include information about: Alternative treatment strategies Amount of time for the procedure The other professionals present during the procedure

To meet the legal standard of informed consent, it should be obtained by the person performing the procedure and should include sufficient information about the risks and benefits of treatment, alternative treatment strategies, and the repercussions of not consenting

Which substance should be removed from TPN if TPN-induced cholestasis develops? Heparin Multivitamins Trace elements

Trace elements (specifically copper and manganese) should be removed from TPN if TPN-induced cholestasis develops.

An infant requiring treatment for syphilis should receive: A single dose of Cefoxitin Seven days of Procaine Penicillin G Ten days of aqueous crystalline Penicillin G

Treatment for syphilis in the newborn is aqueous crystalline Penicillin G 100,000-150,000 U/kg/day divided Q 12 hours over the first 7 days, then divided Q 8 hours thereafter for a total of ten days. Procaine Penicillin G may also be given, 50,000 U/kg/day once a day intramuscularly for 10 days. If greater than one day of therapy is missed, the treatment course must be restarted.

Osteogenesis Imperfecta Type 2

Type 2 i. Low birth weight, Head soft and boggy with large fontanel, Shortened and bowed limbs, Abducted hips, Flattened vertebrae, Blue sclera, Increased mortality rate in first month of life

Osteogenesis Imperfecta Type 3

Type 3 i. Develops within the first 2 years of life, Growth deficiency prenatally, Short stature, Kyphoscoliosis, Multiple fractures, Hearing loss, Triangular facies, Macrocephaly, Dentinogenesis imperfecta

A Type I error is: Rejection of the null hypothesis when it is true A result of Delphi-technique use Acceptance of the null hypothesis when it is false

Type I error is made when the null hypothesis is rejected when it is true, and the probability of committing a Type I error is expressed in the level of significance.

Osteogenesis Imperfecta Type 4

Types 4-6 i. Less severe, Mildly affected stature, Later presentation of fractures, Dentition not commonly affected, Mildly affected sclera

Unconjugated bilirubin is primarily: Fat soluble Water soluble Water and fat soluble

Unconjugated bilirubin is fat soluble, and is insoluble in water. It must become water soluble to be excreted into the bile

Which of the following bilirubin levels would be most concerning in a 37-week infant? Cord bilirubin level of 6 mg/dL at birth Serum bilirubin level of 6 mg/dL at 18 hours of age Serum bilirubin level of 13 mg/dL at 60 hours of age

Unconjugated bilirubin is transported from the fetal to maternal circulation by the placenta for elimination. An elevated cord bilirubin indicates excessive bilirubin production.

The type of Penicillin which can be safely given IV to a neonate is: Procaine Penicillin G Benzathine Penicillin G Aqueous crystalline Penicillin G

Use only aqueous crystalline penicillin G for IV administration. Both Pr0caine Penicillin G and Benzathine Penicillin G may only be given IM

Hemophilia presenting in the neonatal period is due to a deficiency in Factor(s): VIII or X VIII or IX V or XI

VIII and IX

A study comparing the effects of verbal and tactile stimulation on physiologic arousal in infants with congenital heart disease is reported in the literature. What is the independent variable in this study? Physiologic arousal Verbal and tactile stimulation Congenital heart disease

Variables are those concepts that are of interest to the researcher. They include the concepts under investigation that are to be manipulated by the researcher (the independent variables) and those that are dependent on the effect (the dependent variables).

Septic arthritis is: Diagnosed by joint aspiration Most often a primary infection Treated with 10 days of IV antibiotics

While patient exam and x-ray may support the diagnosis of septic arthritis, definitive diagnosis requires aspiration of the affected joint, with the causative organism identified in approximately 60% of cases. It may require 2 - 6 weeks of therapy

Ninety percent of all intraventricular hemorrhages can be diagnosed with a cranial ultrasound at: 3 days of age 5 days of age 7 days of age

While serial ultrasounds can assist with timing of the hemorrhage, a single scan at the end of a week will show 90% of all hemorrhages and the maximum extent of the hemorrhage

Gonococcal conjunctivitis in a jaundiced infant is treated with: Cefoxitin Cefotaxime Ceftriaxone

While the treatment of choice for gonococcal conjunctivitis is a single dose of IM or IV Ceftriaxone, if there is hyperbilirubinemia, a single dose of Cefotaxime may be used

A neonate with elfin facies, supravalvar aortic stenosis and hypercalcemia has chromosomal testing done which reveals a microdeletion of chromosome 7 consistent with Williams syndrome. His mother asks the NNP what this means for her baby's development and intelligence. Your response is based on the knowledge that with this disorder there is: Significant risk of cognitive and developmental delay Mild motor delays are common with no impact on cognition No associated impact on cognitive development

Williams syndrome is associated with s significant risk of cognitive (75%) and developmental (90%) delays. The average IQ is 56.

Meta analysis

a procedure for statistically combining the results of many different research studies

therapeutic window

a range in where the dose of a drug will produce the desired effect with minimal toxicity

Teratogens: Retin A

abortions, cardiac, neuro, renal

Pharmacokinetics

absorption, excretion, metabolism, distribution WHAT THE BODY DOES TO THE DRUGS • Pharmacokinetics is the study of drug movement throughout the body • Pharmacokinetic processes determine the concentration of a drug at its sites of action, and thereby determine the intensity and time course of responses. • Pharmacokinetics also includes • Drug metabolism and drug excretion • Passage of drugs across membranes

cysteine helps with

acid base balance (it is an acid) also helps with calcium and phosphorus in LBW

dilantin and phenobarb are what kind of drugs

acidic

Long term dexamethasone use can suppress which hormone/

adrenocorticotrophic hormone

Malformation

an intrinsically abnormal developmental process, with the early development of a tissue or organ being arrested, delayed or misdirected 1. Cleft lip/palate, cardiac defects, abdominal wall defects, pre-auricular ear pits, hair whorls, clinodactyly

On an electrocardiogram (EKG) strip, the p wave represents: Atrial depolarization Ventricular depolarization Atrial repolarization

atrial depolarization

By week 24 of gestation the major site of hematopoiesis is the: Liver Thymus Bone marrow

bone marrow

Rett syndrome

brain disorder that occurs almost exclusively in girls: 6 to 18 months of apparently normal development before developing problems with language and communication, learning, coordination, microcephaly, scoliosis - ranging in severity

Magnesium and phosphorus levels that are abnormal

can cause an increase in calcium or the calcium will never normalize

Maternal sedatives

can cause respiratory depression and decreased peristalsis in fetus and the levels are usually higher in the fetus

Galactosemia diet

cannot have galactose cannot breastfeed

Teratogens: ace inhibitors

captopril oligo, skull hypoplasia, limb deformities

Teratogens: Valproic acid

cardiac and NTD

why do you have to change an IV dose of medication when switching to oral?

change in bioavailability

Teratogens: phenobarbital

cleft

Teratogens: PHenytoin

cleft lip/palate, nail hypoplasia, cardiac

In this question, a newborn female is born with hydrocephalus (enlarged head), intracranial calcifications, and chorioretinitis (redness and excessive tearing of the eyes). This triad suggests

congenital toxoplasmosis

Teratogens: methotrexate

cranial dysplasia, low ears, flat nasal bridge

Teratogens: carbamezepine

craniofacial and NTD

clearance

degree of efficiency a drug is removed from the body over time

if polyhydramnios is present in an otherwise normal pregnancy....assume

diabetes

peak level

drawn after the dose is given if it is high, decrease the dose

trough level

drawn prior to the dose if it is high, extend interval

IUGr has a

drop in phosphorus levels, refeeding syndrome, hyperglycemia, and a drop in K and M

Pharmacodynamics

drug at the site of action what the drug does to the body, clinical effects, toxic traits of drug the activity of the drug at the receptor site

An edematous infant will have what effect on distribution of a given drug?

edema, more fluid, more TBW, more time to get to organ so you needed a higher dose it has a decreased rate of distribution

The suspicion that a neonate may be septic is increased with an: Elevated lymphocyte count Absolute neutrophil count of 15,000 in the first 24 hours Increased immature to total (I:T) ratio

elevated IT

What needs double coverage with a beta lactam and aminoglycoside?

enterobacter, citro, serratia, pseudomonas

Teratogens: Lithium

epsteins anomaly, seizure, goiter

Other than the skin, organs affected by incontinentia pigmenti include the central nervous system and: Eyes Heart Gastrointestinal system

eyes an X-linked dominant disorder that affects the skin, skeletal system, eyes, and central nervous system.

Preauricular skin tags are most often: Associated with major malformations Can have a malignant transformation Familial

familial

Copper

for RBCs and blood and iron formations give less with biliary problems

Selenium

for antioxidant activities

Manganese

for bones

Carnitine

for fatty acids transfer routine for <1500g

Anilline dye

from fresh cloth diapers, results in cyanosis

If a baby comes in with an E. coli infection think....

galactosemia

drugs with narrow therapeutic range

gentamicin, vancomycin, amikacin

if you have a low alk phos

get a zinc level

General anesthesia

get the baby out in 10 min or else respiratory depression causes hypotension and fetal hypoxia

for a long half life,,,,,

give a loading dose then a maintenance dose to get a steady state

How are aminoglycosides eliminated from the newborn baby?

glomerular filtration

Do no give indomethacin with a

glucocorticosteroid (risk for SIP)

Listeria is a

gram positive rod give amp and gent

Sequence

group of anomalies rising from an initial malformation 1. Potters sequence, Pierre-Robin sequences, Holoprosencephaly sequence

Fetal hemoglobin has an affinity for oxygen that is: High Low The same as adult hemoglobin

high

you do not want to use

highly protein bound drugs

A potential toxic side effect of povidone-iodine use on a preterm newborn's skin is: Hypothyroidism Methemoglobinemia Neurotoxicity

hypothyroidism

A complete or partial lack of oxygen in tissue(s) of the body describes: Asphyxia Hypoxia Ischemia

hypoxia

Volume of distribution is increased for a preterm infant due to

increased extracellular fluid

The most sensitive test to detect hereditary spherocytosis is the

incubated osmotic fragility test.

Hydrochlorathiazide

inhibits sodium reabsorption in distal tubule 2-3mg/kg/day

Vomiting, colicky pain, and bloody or red "currant jelly" stools are symptoms of which of the following? Intussusception Malrotation Necrotizing enterocolitis (NEC)

intussusception

Treatment of Chlamydia trachomatis infections in neonates, including conjunctivitis and pneumonia

is usually oral erythromycin, which is the first line. Oral azithromycin is an acceptable alternative.

Adult erythropoietin (EPO) is produced in the: Liver Spleen Kidney

kidney

gram negative rods (higher mortality)

klebsiella, pseudomonas, proteus, serratia

Varicella immune globulin should be considered for post-exposure prophylaxis in some cases

like women during pregnancy. It is usually given to people who have a high risk of developing severe disease, who are lacking evidence of immunity or who are ineligible for the vaccination.

Epidural

little passage to fetus may cause maternal hypotension* and fetal bradycardia mother may have a temp if used >4hrs

Lasix

loop diuretic 1-2mg/kg/dose electrolyte instability ototoxicity nephrocalcinosis

Management of the infant with pulmonary hypertension is aimed at: Lowering the pulmonary vascular resistance Lowering the systemic vascular resistance Increasing the pulmonary vascular resistance

lowering PVR

Congenital AV block is associated with which of the following maternal diseases? Hypertension Diabetes Lupus erythematous

lupus

zinc is a cofactor for

many enzymes

lipid soluble move easier across

membranes

Which of the following is not a complication of chronic Lasix therapy? Nephrocalcinosis Metabolic alkalosis Metabolic acidosis

metabolic acidosis

increased TBW>

more drug needed

The interplay between genetic and environmental elements and gene-gene interaction is: Multifactorial inheritance Mendelian inheritance Non-Mendelian inheritance

multifactorial

Teratogens: DES

multiple deformities

Magnesium Sulfate

muscle relaxant calcium antagonist rapidly transferred across placenta causes decreased FHR and decreased fetal breathing Neonatal effects: resp depression, hypotonia and apnea, decreased peristalsis

Teratogens: caffeine

mutagenic

Teratogens: Warfarin

nail hypoplasia, seizures, mental deficiency, stippled bone epiphyses, flat nasal bridge

active transport

needs energy, pulls medication into intracellular space

facilitated diffusion or carrier protein

needs something to carry

Infantile acne may develop due to a sebaceous gland response to: Maternal estrogen Abnormal bacterial colonization Neonatal androgens

neonatal androgens

Chlorathizide (diuril)

non calcium wasting

The most common cause of numeric abnormalities of a single chromosome is: Duplication Translocation Nondisjunction

nondisjunction

An infant diagnosed with cutis laxa has surgery for an inguinal hernia. Wound healing in this patient will most likely be: Normal Delayed Complicated by dehiscence

normal

Epstein pearls

oral version of milia; on midline palate or alveolar ridge

Blinded study

people do not know to which group - control or experimental - individual subjects have been randomized. If only subjects have been "blinded" the trial is referred as single blind. If the researchers as well are kept in the dark, the trial is referred as double blind. Double blind trials are more objective. Blinding is done to minimize the impact of personal bias

painless vaginal bleeding after 20 weeks

placental previa

what laboratory values to look at with indomethacin

platelets can cause low platelets

Aldactone

potassium sparing 1-3mg/kg/day

Chromium

potentiates insulin

Factors that decrease metabolism

prematurity decreased CO renal insufficiency immature liver and immature CP450 complex

normal kcal

preterm 110-130kcal/kg/day term: 105-115kcal/kg/day

The most important consequence of drug metabolism is

promotion of renal drug excretion.

you will have an increase amount of free drugs if you do not have enough

protein for them to bind to

Erythromycin orally can cause

pyloric stenosis

The presence of visible peristaltic waves in the epigastrium and a palpable "olive" suggest: Intestinal obstruction Lactobezoar Pyloric stenosis

pyloric stenosis

What is the most reliable way to objectively assess drug therapies

randomized controlled trials

Maternal opioids

rapidly transferred to fetus and can cause respiratory depression

The inheritance pattern for albinism is: Autosomal recessive Autosomal dominant X-linked

recessive

the hydrogen ion is the

regulator

Indocin

renal dysfunction GI perf decreased platelets IVH protection

Ibuprofen

renal dysfunction but less than indomethacin GI perf

Vancomycin and aminoglycosides are eliminated

renally

The binding of albumin to unconjugated bilirubin is: A slow process Permanent Reversible

reversible

A pattern of multiple anomalies derived from a single known cause is a/an: Sequence Mendelian pattern Association

sequence

hexachlorophene

skin disinfectant to prevent staph

Ions to work with in TPN equation

sodium + potassium - phosphorus

anion gap

sodium - chloride + bicarb

most common nonsocomial infection

staph epi

What is the most common type of seizure seen in premature infants? Myoclonic seizures Subtle seizures Tonic seizures

subtle

Systematic Review

summarized findings from multiple studies of a specific clinical practice question or topic that recommend practice changes and future directions for research; one of the strongest sources of evidence for evidence-based practice

topical steroids

systemic issues with growth and adrenal failure

Reglan can cause

tardive dyskinesia

Teratogens: tetracycline

teeth discoloration

steady state

the equilibrium point where amount of drug in equals amount of drug excreted

The more selective a drug is

the fewer side effects it will produce

Thanatophoric dysplasia

the most common lethal skeletal dysplasia characterized by a cloverleaf skull with frontal bossing and hydrocephalus midfacial hypoplasia, short limbs, small thorax (pear shaped)

Teratogens: salicylates

thrombocytopenia, bleeding, PDA closure

betadine causes

transient hypothyroidism

fluconazole

transient liver dysfunction

Infants do not tolerate being overheated and compensate by

vasodilation

Noonan Syndrome main presentation

widely set eyes, low-set ears, short stature, and pulmonic stenosis. Distinct features include hypertelorism, low-set ears, blue irises, ptosis, mild neck webbing, high forehead, down-slanting palpebral fissures, and epicanthic folds.

When a child is found to have myelomeningocele at birth, most (~95%) are associated

with Chiari II malformation

The inheritance pattern of the most common form of ichthyosis in the newborn period is: Autosomal dominant Autosomal recessive X-linked

x-linked

The inheritance pattern for Hemophilia (either Type A or Type B) is: X linked dominant Autosomal recessive X linked recessive

x-linked recessive

Selectivity

• A drug elicits ONLY the responses for which it is given • There is no such thing as a wholly selective drug • All DRUGS CAUSE SIDE EFFECTS

Metabolism

• Also known as biotransformation • Defined as the enzymatic alteration of drug structure • Most often takes place in the liver • Most drug metabolism that takes place in the liver is performed by the hepatic microsomal enzyme system, which is also known as the P450 system • Metabolism does not always result in a smaller molecule • It is important to appreciate that cytochrome P450 is not a single molecule entity but rather a group of 12 closely related enzyme families. Three of the cytochrome P450 (CYP) families - designated CYP1, CYP2 and CYP3 - metabolize drugs. The other 9 families metabolize endogenous compounds (steroids, fatty acids).

Excretion

• Defined as the removal of drugs from the body • Drugs and their metabolites can exit the body through urine, sweat, saliva, breast milk, or expired air • Steps in renal drug excretion • Glomerular filtration • Passive tubular reabsorption • Active tubular secretion • Factors that modify renal drug excretion • pH-dependent ionization • Competition for active tubular transport • Age (newborn, elderly)

Components of pharmacodynamics

• Dose-response relationships • Drug-receptor interactions • Drug responses that do not involve receptors • Interpatient variability in drug responses • The therapeutic index

bacteriocidal

• Drugs are directly lethal to bacteria at clinically achievable concentrations • Aminoglycosides, Beta-tactams, Vancomycin, Quinolones, Rifampin, Metronidazole

bacteriostatic

• Drugs can slow bacterial growth but do not cause cell death • Chloramphenicol, Erythromycin, Clindamycin, Sulfonamides, Trimethoprim, Tetracyclines

Absorption

• Movement of a drug from its site of administration into the blood • The rate of absorption determines how soon effects will begin • The amount of absorption helps determine how intense the effects will be • Factors affecting drug absorption • Rate of dissolution • Surface area • Blood flow • Lipid solubility • pH partitioning

Distribution

• Movement of drugs from the systemic circulation to the site of drug action • Determines by tree major factors • Blood flow to tissues • Most of the tissues are well perfused • Abscesses and tumors • Drug ability to exit the vascular system • Most drugs do not produce their effects within the blood • Important determinant of drug actions • Exiting vascular system is necessary to undergo metabolism and excretion • Entering cells • Drugs' sites of action • Sites of metabolism and excretion

P-glycoprotein

• P-glycoprotein found in the liver, kidney, placenta, intestine, and brain capillaries and can transport a variety of drugs OUT of cells • Coded by the MDR1 gene • Transports lopophilic and cationic compounds • Restricts drug transit across the blood brain barrier

Three ways which drugs cross cell membranes

• Passage through channels or pores - very few drugs can cross since the channels are extremely small and specific for certain molecules. Only smallest compounds can use this way (example: Na or K) • Passage with aid of a transport system - very selective and depends on a drug's structure • Direct penetration of the membrane itself - most common

Maximal efficacy

• The largest effect that a drug can produce (height of the curve) • Match the intensity of the response with the patient's need • Very high maximal efficacy is not always more desirable

Blood-brain barrier

• Tight junctions between the cells that comprise the walls of most capillaries in the central nervous system • Drugs must be able to pass through the cells of the capillary wall • Only drugs that are lipid soluble or that have a transport system can cross the blood-brain barrier to a significant degree


Kaugnay na mga set ng pag-aaral

Chapter 5: Business Communication

View Set

Module 2: North American Geography

View Set

Investigator Obligations in FDA-Regulated Research Quiz

View Set